1200 Board Questions Test 1

Pataasin ang iyong marka sa homework at exams ngayon gamit ang Quizwiz!

10. A 43-year-old otherwise healthy male presents with a 1 week history of a productive cough, subjective fevers and chills, and mild shortness of breath, without chest pain. The patient was transferred from an urgent care facility, where he was found to be hypoxic, with a SaO2 of 87% on room air (RA). A chest x-ray confirms pneumonia. Antibiotics are ordered and the patient is admitted. An admission electrocardiogram (EKG) is ordered and is shown in Figure 1-2. Which of the following is most likely? (A) The limb leads have been reversed. (B) The patient is experiencing demand-ischemia due to his pneumonia causing an ST-elevation myocardial infarction (STEMI) and needs an emergent heart catheterization. (C) An echocardiogram would likely reveal significant underlying structural heart disease. (D) The patient has an increased risk for sudden cardiac death. (E) The patient's calcium level is profoundly decreased. EKG shows pseudo RBBB and presistent ST elevation in V1-3

10. Answer D. This patient's EKG reveals classic or "type 1" Brugada syndrome. Brugada syndrome is characterized by a pseudo-RBBB and persistent ST elevation in V1-V3. There are three unique patterns of ST elevation consistent with Brugada syndrome. In type 1, the elevated ST segment is convex facing upward, and gradually descends to an inverted T wave. This is referred to as a "coved type" Brugada pattern, and is most common and only one really that is potentially diagnostic . This ECG abnormality must be associated with one of the following clinical criteria to make the diagnosis: Documented ventricular fibrillation (VF) or polymorphic ventricular tachycardia (VT). Family history of sudden cardiac death at <45 years old . Coved-type ECGs in family members. Inducibility of VT with programmed electrical stimulation . Syncope. Nocturnal agonal respiration.Types 2 and 3 have identical patterns, in which the elevated ST segment (≥1 mm in type 2, <1 mm in type 3) first descends and then rises again after nearing the baseline, creating a "saddle back" appearance. It is associated with an upright or biphasic T wave. These are less diagnostic but still warrant an investigation. Brugada syndrome is not normally associated with any structural abnormalities. Most "standard" cardiac tests, including echocardiography, stress testing, and cardiac MRI are unrevealing. However, patients are at a much increased risk for sudden cardiac arrest due to ventricular arrhythmias. Unlike patients with hypertrophic obstructive cardiomyopathy, sudden cardiac arrest is not typically due to exercise and more commonly occurs in sleep. Most patients with Brugada syndrome undergo electrophysiology testing to determine their risk for cardiac arrest as well as the need for an automated implantable cardioverter-defibrillator. Due to a mutation in the cardiac sodium channel gene, "sodium channelopathy." Over 60 different mutations have been described so far and at least 50% are spontaneous mutations, but familial clustering and autosomal dominant inheritance has been demonstrated. ECG changes can be transient with Brugada syndrome and can also be unmasked or augmented by multiple factors: fevers, multiple drugs (once that block Na channel like flecainide, CCB, B blockers...), HypoK, HypoTherm and post DC conversion. Only proven therapy is an implanted ICD

30. A 74-year-old man with a history of hyperlipidemia is brought in by emergency medical services (EMS) with an acute ischemic right hemispheric stroke. Soon after returning from CT, he has a generalized seizure which terminates without treatment after 1 minute. Which of the following is true about this patient? (A) The patient should have been treated with prophylactic phenytoin as soon as the diagnosis of ischemic stroke was made. (B) The patient should be given a loading dose of phenytoin after his seizure. (C) Status epilepticus occurs more commonly in the setting of ischemic strokes than in other stroke syndromes. (D) Phenytoin is contraindicated in patients with ischemic stroke due to its potential for causing ataxia. (E) Although isolated seizures are common in patients with ischemic strokes, treatment with antiepileptic drugs is unnecessary because recurrence is uncommon.

30. Answer B. The incidence of seizures after ischemic stroke is uncertain but is cited to be as high as 13%. Typically, patients with seizures after stroke are divided into patients who have seizures within 7 days (early seizures) and those who develop seizures after 7 days (late seizures). Prophylactic use of anticonvulsants in patients with ischemic stroke has not been shown to reduce either early or late seizures although there is limited data on this topic. Therefore, the prophylactic use of anticonvulsants in such patients is not recommended. When seizures do occur, their management is the same as "conventional" seizure management, and status epilepticus is rare. Current guidelines recommend that patients with an ischemic stroke in the ED who develop seizures warrant standard treatment (if necessary) to terminate the seizure followed by treatment with an anticonvulsant to prevent recurrence. There are no special contraindications to antiepileptic use in stroke patients.

31. Which of the following is true regarding genital herpes simplex infection? (A) The primary attack is usually more severe than recurrent episodes. (B) Lesions are almost always painless. (C) The Tzanck smear has >90% sensitivity and specificity. (D) Vesicles should be unroofed to allow drainage of fluid. (E) Antibiotics are indicated during outbreaks to prevent bacterial superinfection.

31. Answer A. Herpes simplex virus (HSV) type 2 causes most genital herpes infections. Primary HSV infection is almost always more severe than recurrences. Outbreaks can occur with any systemic or local stress. The lesions are usually extremely painful and irritating and should never be drained or unroofed as this will inoculate other sites. The sensitivity and specificity of the Tzanck smear are both <80% and viral culture is the gold standard for diagnosis. Antibiotics should not be used in the absence of clear signs of bacterial superinfection. Treatment of HSV outbreaks is with acyclovir, famciclovir, or valacyclovir. Continuous treatment may be necessary to prevent outbreaks in susceptible individuals.

33. A 45-year-old woman with a history of asthma presents with her typical acute asthma exacerbation. She has had a runny nose and a sore throat but denies fever or productive sputum. Examination reveals bilateral wheezes. Her vital signs are 98.6°F, 110, 24, 156/95, 99% RA. She feels slightly better after an albuterol nebulizer treatment. You send off labs, and the white blood cell (WBC) count returns elevated at 13.5K. Which of the following is the next best course of action? (A) Treat community-acquired pneumonia (CAP) with azithromycin (B) Treat CAP with moxifloxacin (C) Treat airway inflammation with prednisone (D) Obtain arterial blood gas sample (E) Observe for 2 more hours, then discharge with outpatient follow-up

33. Answer C. The patient has a typical acute asthma exacerbation. The mainstays of treatment are bronchodilators for bronchospasm and corticosteroids for airway inflammation. Despite her upper respiratory symptoms and the presence of leukocytosis, she has no signs of superimposed pneumonia, so empiric treatment is not indicated. A chest x-ray could be obtained to further evaluate the possibility of pneumonia. Arterial blood gas sampling is only helpful in patients who are deteriorating despite aggressive bronchodilator therapy and should not routinely be obtained in the evaluation of most acute asthma exacerbations. Patients with asthma exacerbations should not be discharged without providing either systemic or inhaled corticosteroid therapy.

37. A 6-year-old girl presents with abdominal pain. She has had moderate, constant periumbilical pain for several hours with associated nausea. Her parents noted a rash on her legs and buttocks for several days, which they attributed to poison ivy. Physical examination demonstrates an afebrile, uncomfortable patient, diffuse abdominal tenderness without true rebound or guarding, a maculopapular rash on the legs and buttocks, and diffuse joint tenderness. Which of the following is the most important next step in evaluation? (A) Urinalysis (B) CT scan of the abdomen/pelvis (C) Anti-streptolysin O antibodies (ASO) titer (D) Blood cultures (E) Meckel scan

37. Answer A. The patient has evidence of Henoch-Schonlein purpura (HSP), an immune-mediated vasculitis of idiopathic origin. Young children are the highest risk group. Dermatologic, gastrointestinal, renal, and musculoskeletal findings are seen. The rash of HSP is characteristic: A maculopapular eruption on the legs and buttocks and almost never involving the upper extremities or trunk. Abdominal pain is due to intestinal vasculitis or intussusception, which occurs with higher frequency in patients with HSP than the normal population. Renal involvement may be due to glomerulonephritis, which is detected as hematuria on urinalysis. In addition, patients may have mild to nephritic range proteinuria. While the long-term prognosis of children with renal involvement is good, persistent renal dysfunction accounts for a significant portion of the morbidity associated with HSP. Current management of HSP involves potential administration of corticosteroids or intravenous immunoglobulin (IVIG) to prevent and treat glomerulonephritis. Abdominopelvic CT scan is not indicated, as HSP is a clinical diagnosis. Despite the possibility of HSP as an immune-mediated response to streptococcal infection, ASO titers are not routinely indicated, as they do not change management. Blood cultures are not necessary in these patients except when systemic infection is suspected by fever and focal abdominal tenderness. A Meckel scan is used to detect the presence of Meckel diverticulum, which usually presents with painless rectal bleeding rather than the constellation of signs seen in HSP.

A 44-year-old woman presents with right wrist pain. She fell on her hand the previous day and has pain in her radial wrist. The wrist is tender in the anatomic snuffbox. Plain radiographs of the wrist are completely normal. Which of the following is the most appropriate next step in management? (A) Orthopedics consultation (B) Discharge home with thumb range of motion exercises (C) Discharge home with Velcro wrist splint (D) Discharge home with thumb spica splint (E) Admit for observation of wrist

4. Answer D. Patients with fall on outstretched hand (FOOSH) injuries may have bony or ligamentous damage in any part of their upper extremity. The bones of the wrist are most susceptible to injury. The anatomic snuffbox is demarcated by the extensor pollicis brevis and longus tendons just proximal to the thumb and tenderness in this region indicates possible fracture to the scaphoid. Scaphoid fracture is particularly dangerous because of the high rate of AVN and resulting limitation of thumb function. Almost one-fifth of all scaphoid fractures are invisible on acute radiographs, so appropriate management in suspected cases includes immobilization in a thumb spica cast and repeat radiographs in 1 to 2 weeks. There is no need for emergent orthopedic consultation in patients with negative x-rays. Discharging the patient home without a splint that immobilizes the thumb is contraindicated. Admission for observation is not an appropriate use of resources.

40. What is the most common cause of death in patients with hemophilia A? (A) Septic shock (B) Myocardial infarction (MI) (C) Gastrointestinal bleeding (D) Intracranial hemorrhage (E) Congestive heart failure

40. Answer D. Hemophilia A occurs with a genetic deficiency in factor VIII, predisposing the patient to hemorrhagic complications. The most common cause of death is from head trauma causing massive intracranial hemorrhage. Treatment of bleeding episodes in patients with hemophilia A is with factor VIII. Every patient with hemophilia A should receive factor VIII after head trauma (whether or not there is evidence of intracranial bleeding), as the risk of delayed bleeding is high. Sepsis and MI occur with similar incidence to the general population. Gastrointestinal bleeding does occur in hemophiliacs but is not as common a cause of morbidity or mortality as intracranial bleeding. Congestive heart failure may be secondary to high-output failure caused by anemia but is usually a more chronic process and is not primarily responsible for death.

41. What is the half-life of carboxyhemoglobin with a 100% oxygen nonrebreather mask? (A) 6 hours (B) 3 hours (C) 90 minutes (D) 60 minutes (E) 30 minutes

41. Answer C. The half-life of carboxyhemoglobin is 6 hours on room air, 90 minutes on 100% nonrebreather, and 30 minutes on hyperbaric oxygen.

42. What is the most common cause of lower gastrointestinal (GI) bleeding (LGIB) in children? (A) Anal fissure (B) Hemorrhoids (C) Henoch-Schönlein purpura (D) Food allergy (E) Meckel's diverticulum

42. Answer A. Anal fissures are the most commonly encountered anorectal problem in children and they are the most common cause of LGIB in this population, particularly in the first 2 years of life. Meckel's diverticulum is the most common cause of substantial GI bleeding in this population.

49. A 65-year-old man with a past history of severe hypertension presents with sudden onset of shortness of breath and chest pain that started 8 hours ago. He also reports orthopnea and lower extremity edema but denies chest pain. He saw his primary care physician who noted a new diastolic murmur. Vital signs are as follows: T, 98.4; HR, 110; BP, 115/76; RR, 25; and SpO2, 90% RA. On exam, the patient is in moderate respiratory distress, with a diastolic murmur at the right upper sternal border, bilateral crackles, and lower extremity pitting edema. The electrocardiogram (EKG) shows signs of left ventricular hypertrophy (LVH), troponin I is negative, and chest x-ray demonstrates cardiomegaly and pulmonary edema. Which of the following is the most appropriate definitive therapy? (A) Percutaneous transluminal coronary angioplasty (B) Pacemaker placement (C) Surgical valve replacement (D) Dobutamine (E) Tissue plasminogen activator

49. Answer C. The patient has CHF from acute aortic regurgitation. The history of a new diastolic murmur with signs of pulmonary edema and normal EKG and cardiac markers indicates a valvular etiology. Surgical aortic valve replacement is the only definitive treatment for these patients. The two most common causes of acute aortic regurgitation are aortic dissection and endocarditis. This patient's past history of severe HTN and chest pain suggests dissection as a likely cause. Once the dissection extended to the aortic valve, the patient developed acute, severe AR and CHF that compromised his ability to generate an adequate BP. While his BP appears normal, his history of "severe" HTN suggests that his current pressure is markedly low compared to normal. Clinically, an imaging study to determine the cause of AR would typically be performed before taking the patient to the OR. Choices A and E are appropriate for ST-elevation myocardial infarction (STEMI) and are not indicated here. Choice B is not indicated, as the EKG does not demonstrate dysrhythmia as a cause for the heart failure. Choice D may be indicated in cases of cardiogenic shock but is not a definitive treatment.

52. Retrobulbar hemorrhage can result in what devastating complication? (A) Corneal abrasion (B) Hypopyon (C) Central retinal artery occlusion (CRAO) (D) Hyphema (E) Corneal ulcer

52. Answer C. Retrobulbar hemorrhage is a result of ocular trauma that causes pressure on the posterior portion of the eye. The globe is pushed outward, and proptosis may be seen on physical examination. Increased pressure in the orbit can compress the central retinal artery or vein and cause loss of vision. Choices A, B, D, and E are all pathologic processes involving the anterior portion of the eye and are not usually caused by a retrobulbar hematoma. Secondary glaucoma may occur as a result of increased overall pressure in the globe, including in the anterior chamber. Treatment of a retrobulbar hemorrhage involves emergent lateral canthotomy and drainage of the hematoma out of the temporal border of the globe. Failure to perform lateral canthotomy for acute retrobulbar hemorrhage may result in irreversible vision loss in as little as 90 minutes.

60. Which of the following is true regarding Bordetella pertussis infections? (A) Prophylaxis with erythromycin is recommended for adults who come into contact with pertussis-infected individuals. (B) Almost all cases of pertussis in adolescents and adults occur in previously unvaccinated patients. (C) Immunization against B. pertussis confers lifelong immunity. (D) Older children infected with B. pertussis have the most severe disease. (E) The clinical course in symptomatic adults is characterized by a mild cough that resolves within 3 to 7 days.

60. Answer A. The incidence of B. pertussis infection is rising, with the number of reported cases in the United States increasing sixfold since 1980. This has occurred despite immunization rates of 80% among young children. Most of the increase is due to an increased number of adolescents and adults diagnosed with the disease. Furthermore, it is widely thought that the true scope of the problem is grossly underestimated because of the failure of physicians to recognize the illness, as well as their failure to report the illness when it is diagnosed. Almost all cases of pertussis in adolescents and adults occur in patients who have been previously vaccinated or in patients who have been previously infected with B. pertussis. Contrary to popular belief, neither B. pertussis infection nor vaccination with either the cellular or acellular vaccine confers lifelong immunity. In fact, natural infection with B. pertussis results in approximately 15 years of immunity to reinfection. This is much greater than the 4 to 8 years worth of immunity offered by the vaccines (the acellular vaccine, which is currently used in the United States offers a shorter duration of immunity than the cellular vaccine, roughly from 4 to 6 years). The recommended vaccination schedule in the United States advises that infants be vaccinated at 2, 4, and 6 months of age, with boosters at 18 months and then again between 4 and 6 years. Therefore, most children born in the United States should carry immunity through the ages of 10 to 12 years. Since the most severe illness occurs in children less than 1 year, adolescents and adults have not been offered booster shots beyond childhood. However, because of the rising incidence of recognized disease in adolescents and adults, as well as the likely enormous asymptomatic disease burden in this population, the CDC and the US Advisory Committee on Immunization Practices (ACIP) now recommends that all adults under 65 receive a single acellular booster along with tetanus and diphtheria in Tdap (Boostrix), which is often given in the ED in the setting of trauma (in place of isolated tetanus toxoid). Patients >65 years old receive Adacel, a different Tdap formulation. B. pertussis infection in adults ranges from subclinical infection to a prolonged illness mostly characterized by a nagging, paroxysmal cough, which may interfere with sleep. The mean duration of illness in adults is 36 to 48 days. Erythromycin is the drug of choice for the treatment of B. pertussis infection although azithromycin and clarithromycin have been shown to be equally efficacious with fewer side effects. TMP-SMX is an additional alternative. Erythromycin is also recommended for prophylaxis of individuals who have come into contact with patients who are infected. It is thought to be effective in preventing disease as long as it is given before the onset of symptoms. Owing to the decreased infectivity of B. pertussis as the disease progresses, prophylaxis is generally unnecessary in individuals who come into contact with a patient who has been symptomatic for >3 weeks.

61. A 23-year-old man presents with shoulder pain after falling. A shoulder radiograph is shown in Figure 1-9. Which of the following is the most likely additional injury? (A) Axillary nerve injury (B) Glenoid rim disruption (C) Brachial artery injury (D) Humeral head fracture (E) Biceps muscle tear

61. Answer A. The radiograph demonstrates an anterior shoulder dislocation, with the humeral head displaced anteriorly relative to the glenoid. Common complications include (in order of decreasing frequency) axillary nerve injury (causing deltoid anesthesia), humeral head fracture (Hillman-Sachs deformity), and glenoid rim disruption (Bankart lesion). Vascular compromise in shoulder dislocations is rare. Acromioclavicular separation and clavicular fractures are rarely associated with shoulder dislocations because of different mechanisms. The mechanism of an anterior dislocation is trauma to the abducted and externally rotated upper extremity. Clavicle fractures and acromioclavicular separations tend to occur with trauma to the adducted arm. (Figure courtesy of Shoma Desai, MD. Reprinted with permission from Desai S. Greenberg's Text-Atlas of Emergency Medicine. Lippincott Williams & Wilkins; 2004:497.)

83. What is the most common dysrhythmia in hypothyroid cardiovascular disease? (A) Atrial fibrillation (B) Long QT syndrome (C) Junctional escape rhythm (D) Sinus rhythm with left or right bundle branch block (E) Sinus bradycardia

83. Answer E. The most common dysrhythmia in hypothyroid cardiac disease is sinus bradycardia. Cardiomegaly and depressed cardiac contractility are other manifestations.

77. Which is the most appropriate management for a patient with suspected smallpox lesions? (A) Acyclovir (B) Vaccination (C) Ganciclovir (D) Rifampin (E) Isolation

77. Answer E. Smallpox is due to Variola major, a member of the poxvirus family. It was eradicated in the 1970s and now exists only in laboratories. Mortality of smallpox is almost 30% and contagiousness is extremely high. The only effective strategies for management are prevention, vaccination, and isolation. Vaccination is only effective if given before or within a few days of exposure. No antivirals or antibiotic are effective in management.

14. Common features of atopic dermatitis include which of the following? (A) Pruritus (B) Flexor surface involvement in adults (C) Facial involvement among infants (D) Frequent involvement of the hands in adulthood (hand dermatitis) (E) All of the above

14. Answer E. Atopic dermatitis is a chronic inflammatory skin disease that is characterized by intense pruritus and an eczematous rash. There is no classic lesion in atopic dermatitis. The disease may initially manifest itself with erythematous papules or vesicles, with diffuse erythema and frequently contains a weeping exudate. Over time, dryness, scaling, and lichenification predominate. Although there is no typical lesion, there are many classic features of atopic dermatitis. Pruritus is the hallmark of the disease and is typically intense. In adults, the disease has a predilection for the flexural creases such as the anterior elbow, ankle, and neck as well as the posterior knee. In contrast, in infants the disease is characterized by dry, red, scaly involvement of the cheeks. The chin is also commonly involved and chin inflammation may be more severe due to the added irritation from drooling. Involvement of the hands is extremely common in adults and is frequently exacerbated by occupational exposures.

45. Which of the following is the number one cause of death in patients with congestive heart failure (CHF)? (A) Progressive hemodynamic deterioration (B) Urinary tract infection (UTI) (C) Pneumonia (D) Stroke (E) Pulmonary embolism CT shows convex hyperdense area parietal region

45. Answer A. Patients with CHF most commonly die due to hemodynamic decline, followed by dysrhythmia. Mortality from fluid overload has been reduced by β- blockers and ACE inhibitors, and short-term mortality from dysrhythmias has been reduced by automated internal cardiac defibrillators (AICDs). Choices B, C, D, and E are all important, but are less common causes of death in patients with CHF.

100. A 44-year-old woman presents with right leg pain. She has noticed increasing pain in the middle of her tibia over the last few weeks since she started training for a marathon. Physical examination of the entire lower extremity is normal except for mild tenderness to palpation in the area of pain. Radiographs of the tibia are normal. Which of the following is true regarding this patient's condition? (A) Most cases require surgery for definitive management. (B) The patient may only engage in non-impact exercise. (C) Magnetic resonance imaging (MRI) is the test of choice to differentiate shin splints from stress fractures. (D) Men are at higher risk than women. (E) The metatarsals are the most common bones involved.

100. Answer C. While this patient could have a stress fracture, her presentation is most consistent with shin splints (medial tibial stress syndrome). Like stress fractures, shin splints are due to overuse, and are more common in women than men. While the tibia is the obvious bone involved in shin splints, it is also the most common involved bone in patients with stress fractures (metatarsals are second most common). Radiographs are normal in patients with shin splints, and are most often normal in patients with stress fractures. Subacute radiographs (3-4 weeks) of patients with stress fractures are abnormal up to 50% of the time. Bone scan has much higher sensitivity than radiographs, but a much lower specificity. Therefore, MRIs have become the test of choice to differentiate shin splints from true stress fractures if the diagnosis is unclear. However, advanced imaging is not indicated in the vast majority of cases, as management is rarely affected and the diagnosis is most often clinical. Standard treatment with rest (though patients can continue to run at reduced mileage), ice, and nonsteroidal anti-inflammatory drugs (NSAIDs) results in resolution of almost all cases of shin splints syndrome. Stress fractures require more prolonged rest, participation in non-impact activities only, and a monitored, more graduated return to running.

11. Among adults with asthma, which of the following demographics has the highest risk of morbidity and mortality? (A) White men (B) White women (C) Black men (D) Black women (E) Asian men

11. Answer D. Asthma hospitalization and mortality rates are highest for Puerto Ricans and African Americans. Western countries have the highest rates of asthma, underscoring the role that urbanization plays in the development of this disease. It is unclear why females have higher rates of morbidity and mortality than males. Proposed factors include exacerbation of asthma symptoms by estrogen and bronchial hyper-responsiveness to triggers such as cigarette smoking. Among children, asthma carries greater morbidity and mortality among boys than girls.

12. The most common cause of acute mesenteric ischemia is: (A) Thrombosis in the superior mesenteric artery (SMA). (B) Mesenteric vein thrombosis. (C) Nonocclusive mesenteric vascular disease. (D) SMA embolism. (E) Abdominal aortic aneurysm involving the SMA.

12. Answer D. Emboli in the SMA account for 50% of the cases of acute mesenteric ischemia. Emboli usually originate in the left atrium or ventricle. Most emboli lodge just distal to a major branch point, and greater than 50% of SMA emboli are located just distal to the origin of the middle colic artery. Thrombosis of the SMA and mesenteric vein thrombosis each account for approximately 15% of cases.

13. A 13-year-old boy is brought to the emergency department (ED) by his mother with a 3-week history of a nonproductive cough that "won't go away." She asks you if he might have "walking pneumonia." Which of the following is true? (A) Walking pneumonia may frequently be caused by Chlamydia pneumoniae, a sexually transmitted organism. (B) This is usually caused by Epstein-Barr virus, the same virus that causes "mono," and it may take several weeks for him to improve. (C) The patient should be admitted to the intensive care unit (ICU) for IV antibiotics as walking pneumonia rapidly progresses to sepsis and respiratory failure. (D) The most common causes are Mycoplasma pneumoniae and C. pneumoniae, and macrolides are the antibiotics of choice. (E) If his chest x-ray is normal, then a diagnosis of walking pneumonia is less likely.

13. Answer D. "Walking pneumonia" most frequently refers to "atypical" pneumonia, a term which has been used to distinguish infections caused by M. pneumoniae, C. pneumoniae, and Legionella from infections caused by "classic" bacteria such as S. pneumoniae. The term walking pneumonia came about because patients with this entity are usually well enough to be out of bed and able to perform their normal activities without difficulty. The course tends to be mild although it may be prolonged. Several recent studies have demonstrated that it is not possible to differentiate between typical and atypical causes of pneumonia based on symptoms or chest radiography. Furthermore, chest x-rays may appear normal in either typical or atypical cases. However, because Mycoplasma and Chlamydia are the most common causes of community-acquired pneumonia (CAP) in this patient's age group (5 to 15 years), and because he lacks signs and symptoms suggesting a critical illness, it is reasonable to treat him empirically for "atypical" CAP. Macrolides are the antibiotics of choice for Mycoplasma and Chlamydia.

88. Which of the following is true regarding botulism? (A) It is not contagious. (B) It usually causes an ascending paralysis. (C) It usually spares the cranial nerves. (D) It stimulates presynaptic acetylcholine release. (E) Broad-spectrum antibiotic therapy significantly improves survival.

88. Answer A. Botulism occurs as a result of the toxin released by the anaerobic bacillus Clostridium botulinum. Botulinum toxin is considered the most potent poison known to humans, but it is not transmitted from person to person. Clinical botulism manifests as a descending paralysis, usually involving the cranial nerves. The mechanism is irreversible inhibition of presynaptic acetylcholine release. Antibiotics have little effect on treatment of botulism. Treatment involves supportive care, mechanical ventilation, and antitoxin therapy.

15. A 35-year-old woman presents in a coma (Glasgow Coma Scale 3) after a motor vehicle crash and is intubated for airway protection. Further evaluation reveals no lifethreatening chest, abdomen, or pelvic injuries. Vital signs are normal. A computed tomography (CT) scan of the head is normal. Which of the following is the most likely diagnosis? (A) Epidural hematoma (B) Subdural hematoma (C) Diffuse axonal injury (DAI) (D) Cerebral contusion (E) Intraparenchymal hematoma

15. Answer C. DAI is an important traumatic cause of coma that is not due to a mass lesion or frank intracerebral hemorrhage. Initial CT scan is almost always normal in patients with DAI, but MRI may show diffuse white matter disruption due to axonal fiber injury. Because of the difficulty in gauging DAI on neuroimaging, the prognosis is based totally on clinical parameters. The duration of coma obviously correlates with severity of injury. Patients with DAI who awaken from coma within 24 hours may have few permanent disabilities. Those in coma for longer than 24 hours tend to have much more grim outcomes, including persistent vegetative state or extreme cognitive dysfunction. Most types of acute intracranial hemorrhage severe enough to lead to coma would be evident on initial CT scan, including epidural or subdural hematomas, cerebral contusions, and intraparenchymal hematoma. An important exception is subarachnoid hemorrhage, which may not be visible on CT scan and is a common hemorrhagic cause of altered mental status after trauma

16. An 87-year-old woman is brought to the ED by her caretaker with dehydration and excessive somnolence. Her initial blood work reveals a sodium level of 119 mEq per L and a glucose level of 1,100. Which of the following represents her actual sodium level? (A) 109 mEq per L (B) 119 mEq per L (C) 125 mEq per L (D) 130 mEq per L (E) 135 mEq per L

16. Answer E. Since glucose exerts an osmotic pressure on cell membranes, water is extruded from cells into the intravascular space, thereby diluting plasma sodium. The sodium level must therefore be corrected for hyperglycemia by using the following formula: Corrected Na+ = [Na+] + {1.6 × ([glucose] - 100)/100} While this patient's plasma concentration is at the low end of the normal range, patients with hyperosmolar hyperglycemic syndrome are typically severely sodium deplete. This occurs because sodium is lost in the urine as a result of extensive glycosuria.

17. A 3-year-old girl swallows a button battery. Plain radiographs demonstrate that the battery is lodged in the esophagus. Which of the following is the most appropriate next step in management? (A) Expectant management (B) Endoscopic removal (C) Ipecac for therapeutic emesis (D) Activated charcoal (E) Whole bowel irrigation

17. Answer B. Button batteries lodged in the esophagus or trachea can cause obstruction, necrosis, and perforation. Esophageal button batteries should be removed urgently with upper endoscopy. Any button battery distal to the esophagus may be managed expectantly and will likely pass without any specific treatment. Ipecac is rarely indicated for any ingestion anymore. Activated charcoal is not indicated for foreign body ingestions. Whole bowel irrigation may be useful to help speed passage of postesophageal button battery that has been slow to progress with expectant management alone.

18. A patient presents with the electrocardiogram (EKG) shown in Figure 1-3. Which of the following is the most likely pathophysiologic mechanism? (A) Reentry (B) Increased automaticity (C) Atrioventricular (AV) blockade (D) Preexcitation (E) Infarction EKG shows delta waves

18. Answer D. The EKG demonstrates a preexcitation pattern. Leads V1 through V6 exhibit a gradual sloping of the QRS complex (delta wave) combined with a shortened PR interval. The most common type of preexcitation is Wolff-Parkinson-White (WPW) syndrome. Patients have an accessory conductive pathway from the atria to the ventricles, which preexcites the ventricular myocytes before the AV node releases the normal sinoatrial depolarization. As a result, patients with WPW have a shortened PR interval and a delayed QRS upstroke, called the delta wave. Patients with WPW syndrome can have reentrant dysrhythmias, in which the accessory pathway can either conduct retrograde (in which the AV node conducts in the normal direction, producing a narrow QRS complex and an "orthodromic" pattern) or anterograde (in which the AV node conducts backward, producing a wide QRS complex and an "antidromic" pattern). A patient with WPW syndrome, tachycardia, and wide QRS complexes suggests the presence of an antidromic conduction pattern (where the accessory pathway conducts anterograde and the AV node conducts retrograde). Selective AV nodal active agents are contraindicated in this circumstance, as inhibition of the AV node will cause faster conduction through the anterograde accessory pathway, which is already at high risk for degeneration into an unstable rhythm. The treatment of choice in stable antidromic or irregular tachycardias in patients with WPW is amiodarone or procainamide. Unstable patients require cardioversion.

19. A 46-year-old male suffered a snakebite on his hand by a "brightly colored snake" at a ranch in Texas. His friend snapped a picture of the snake and a colleague notes it looks like a coral snake. The patient has minimal pain and swelling at the bite site. Which of the following is the next best step in management (Fig. 1-4)? (A) The patient should be observed for a minimum of 6 hours and then discharged if he remains asymptomatic. (B) The patient should immediately receive crotalidae polyvalent immune fab ovine (CroFab) antivenin. (C) The wound should be irrigated with sterile saline and a sterile suction catheter to help remove venom. (D) The patient should immediately receive North American coral snake antivenin. (E) The patient should receive an intramuscular mixture of antivenins as close to the bite site as possible. Snake with black/white stripes shown

19. Answer D. The patient was bitten by a coral snake, which is identifiable in North America by the presence of red bands that are bounded by yellow bands ("red on yellow kill a fellow"). In the US, Coral snake bites are much less common than Crotalinae snake bites, and primarily occur in Texas, and Florida. All patients suffering coral snake envenomation should be treated with antivenin even in the absence of clinical symptoms. Coral snake venom is neurotoxic and signs of neurotoxicity may develop rapidly or be delayed for up to 12 hours. In addition, patients may experience nausea, vomiting, headache, and sweating. Ptosis is frequently the initial sign of neurotoxicity and may be followed by delirium, tremors, drowsiness, hypersalivation, and multiple cranial nerve abnormalities (dysarthria, diplopia, dysphagia). In severe envenomations, respiratory muscle paralysis occurs leading to respiratory failure and death. If a pit viper bite was suspected, then observation would be the best course of action. If no signs of envenomation develop 4 to 6 hours after pit viper envenomation, the patient may safely be discharged with routine follow-up instructions. (Figure © Dr. Julian White.)

Which of the following is true regarding adult epiglottitis? (A) Airway obstruction is usually caused by inflammation of the infraglottic tissues. (B) Drooling and stridor are infrequent presenting signs. (C) The disease is more common in winter. (D) Nebulized racemic epinephrine has been shown to decrease the need for intubation. (E) Normal lateral neck x-rays can safely exclude epiglottitis.

2. Answer B. Other than the epiglottis, epiglottitis may involve several supraglottic structures, including the vallecula, aryepiglottic folds, arytenoids, lingual tonsils, and base of the tongue. Inflammation does not extend to the infraglottic tissues because of the robust attachments between the infraglottic mucosa and submucosa. Owing to the variable involvement of several supraglottic structures, epiglottitis is sometimes referred to as supraglottitis. Drooling and stridor are unusual presenting signs in patients with epiglottitis. Historically, however, it has been thought that patients presenting with these symptoms, especially if they have developed over a short time period, are at higher risk for subsequent airway obstruction. No large, prospective trials have been conducted to sort this out. Most often, patients with epiglottitis present with a severe sore throat and painful dysphagia. Adult epiglottitis does not demonstrate any seasonal variation, but appears more common in males and smokers. Neither epinephrine nor corticosteroids has been shown to be beneficial, despite their widespread use. Caution is advised regarding the use of epinephrine as a temporizing measure in patients with epiglottis due to possible rebound upper-airway constriction after the treatment is completed. Ninety percent of patients with epiglottitis will have abnormal lateral neck films. The classic finding is the "thumb" or "thumbprint" sign, indicating the presence of a swollen, inflamed epiglottis. However, a normal film cannot exclude the disease. Direct nasopharyngoscopy has been the gold standard of diagnosis, as it allows direct visualization of the tissue in question. Recently, however, the "vallecula" sign has been suggested as another method of screening for the presence of epiglottitis on lateral neck films. This method relies on the physician's ability to locate the base of the tongue and trace it inferiorly toward the hyoid bone to locate the vallecula. If the vallecula is not deep and roughly parallel to the pharyngotracheal air column, then epiglottitis is present. In a small trial, this sign was shown to be 98% sensitive and 100% specific for epiglottitis.

20. Which of the following studies has the highest sensitivity for traumatic pericardial tamponade? (A) Anteroposterior (AP) chest x-ray (B) Lateral chest x-ray (C) Electrocardiogram (EKG) (D) Focused Assessment with Sonography in Trauma (FAST) scan (E) Diagnostic peritoneal lavage (DPL)

20. Answer D. Pericardial tamponade usually results from a penetrating thoracic mechanism causing cardiac or mediastinal injury and accumulation of blood into the pericardium. Pericardial effusion and tamponade are readily seen on the subxiphoid and parasternal views of the FAST scan. Chest x-ray may show a large, waterbottle heart if the effusion is large enough, but tamponade physiology can occur with small, rapidly accumulating effusions which may be invisible on plain radiography. The EKG finding of electrical alternans due to swinging of the heart throughout the cardiac cycle is highly specific for pericardial effusion, but occurs in less than one-third of cases. DPL is highly sensitive for intraperitoneal injury but has no utility in screening for pericardial effusion.

21. A 52-year-old previously healthy female collapses while coaching her daughter's soccer team and bystanders initiate chest compressions. When EMS arrives, they find the patient in ventricular fibrillation. EMS immediately defibrillates the patient and initiates ACLS. They also placed a temporary supraglottic airway. After following ACLS algorithms for 14 minutes including three defibrillation attempts, the patient experiences a return of spontaneous circulation (ROSC). Shortly after arrival in the emergency department (ED), a definitive airway is obtained and placement is confirmed by x-ray. The patient is noted to be comatose, with a GCS of 3, and has the following vital signs: T 95.8°F, P 115, BP 79/40 (MAP = 53), SaO2 97% on the ventilator. Which of the following is true? A) The patient is not a candidate for therapeutic hypothermia because her GCS score is too low. (B) The patient should undergo immediate cooling with a specialized intravenous cooling catheter. (C) The patient is not likely to benefit from cooling because her core temperature is already low. (D) Vasopressors are needed to raise her blood pressure prior to the initiation of cooling. (E) Shivering is an expected, benign side effect of therapy, and does not require treatment.

21. Answer D. Therapeutic hypothermia is primarily used to mitigate the neurologic devastation that occurs in the setting of cardiac arrest. While the initial trials were conducted in patients with cardiac arrest due to ventricular fibrillation or pulseless ventricular tachycardia, most published guidelines suggest that it can be used after any cardiac arrest thought due to cardiac origin (e.g., PEA or asystole). Emergency physicians should strongly consider using therapeutic hypothermia in all patients with successful ROSC after cardiac arrest who are hemodynamically stable with a MAP >65, yet who are comatose with a GCS <8. While there are no evidence-based guidelines, most guidelines suggest that therapeutic hypothermia should be considered in all patients with ROSC <1 hour after the beginning of resuscitative efforts and should be started within 6 hours. Hypotensive patients in cardiogenic shock may be candidates for therapeutic hypothermia provided their MAP can be stably maintained >65 with the use of vasopressors. The patient described in this vignette (MAP = 53) would require vasopressors prior to the initiation of cooling. Ongoing hypotension is a contraindication for therapeutic hypothermia. Other contraindications to therapeutic hypothermia include patients with a core temperature <30°C (86°F) upon arrival, patients with poor baseline mental status, terminally ill patients, pregnancy (relative), traumatic arrest (relative), and patients with inherited clotting disorders. Shivering is an expected and common side effect of therapy but is not benign. Shivering leads to increased oxygen consumption, an elevated heart rate, increased work of breathing, and a generalized increase in the stress response. The heat generated from shivering can also impede cooling. Shivering should be managed with analgesia and sedation, with paralytics used if all other methodologies fail.

22. A 29-year-old woman presents after going running on the treadmill with cough, shortness of breath, and chest tightness. She has no throat swelling or rash. Her examination is normal except for bilateral wheezes. She has been taking an over-the-counter medication for her intermittent migraine headaches. What medication is the most likely cause? (A) Acetaminophen (B) Phenylephrine (C) Naproxen (D) Capsaicin (E) Paracetamol

22. Answer C. The patient likely has aspirin-exacerbated respiratory disease, which is caused by aspirin and other NSAIDs. NSAIDs inhibit cyclooxygenase, reducing the formation of prostaglandin E2 (PGE2) from arachidonic acid. PGE2 normally serves to inhibit formation of inflammatory leukotrienes. With the formation of PGE2 reduced, leukotrienes are increased, triggering airway inflammation and bronchospasm. Exercise can trigger these bronchospastic attacks. Of the answer choices, only naproxen is an NSAID. Phenylephrine is an alpha-1 adrenergic agent with little effect on airway physiology. Capsaicin is a topical agent that reduces accumulation of substance P in peripheral sensory neurons. Paracetamol is the formulation of acetaminophen that is widely used in the United Kingdom.

23. A 23-year-old woman presents with a fever and sore throat for 3 days. She thinks she has strep throat. Which of the following findings is more characteristic of streptococcal pharyngitis than viral pharyngitis? (A) Runny nose (B) Odynophagia (C) There are no reliably distinct features (D) Conjunctivitis (E) Bilateral tonsillar enlargement

23. Answer C. Both viral and group A streptococcal pharyngitis usually cause odynophagia and tonsillar enlargement. The absence of cough and presence of tender anterior cervical lymphadenopathy are more common with bacterial pharyngitis. Upper respiratory symptoms and conjunctivitis in conjunction with pharyngitis and fever are most often viral, usually due to adenovirus. However, most patients do not present with a unique clinical picture. Instead, most infectious agents cause a similar clinical syndrome, which is nearly impossible for clinicians to differentiate without testing. Reliance on the Centor criteria results in unnecessary treatment nearly 33% of the time. The sensitivity of older rapid antigen testing for streptococcal pharyngitis is reported to be approximately 80%, compared with 90% to 95% for throat culture. Newer rapid antigen tests are reported to have comparable sensitivities to throat culture, potentially eliminating delays in care.

24. A 22-year-old man presents with palpitations. He reports no chest pain, shortness of breath, or lower extremity edema. He states that he "got really drunk" the night before. He denies any past medical history, family history, or illicit drug use. His examination is unremarkable except for irregular tachycardia. His electrocardiogram (EKG) is shown in Figure 1-5. Which of the following is the most likely etiology? (A) Myocardial infarction (MI) (B) Pulmonary embolism (C) Alcohol use (D) Hypertension (E) Diabetes

24. Answer C. The EKG demonstrates atrial fibrillation with rapid ventricular response. The most likely cause in this young adult with no medical history except for alcohol abuse is "holiday heart syndrome," which can occur within 2 days of an alcohol binge. It often resolves spontaneously but may require rate control therapy and possibly anticoagulation. In the absence of cocaine use, MI would be extremely unlikely in an otherwise healthy 22-year-old. Pulmonary embolism is an important cause of atrial fibrillation but is not likely in the absence of chest pain, dyspnea, or risk factors. The physical examination is unremarkable, so hypertension is unlikely. Diabetes does not confer an increased risk of atrial fibrillation.

25. Which of the following is true regarding pediatric community-acquired pneumonia (CAP)? (A) The most common cause of pneumonia in the neonate is Mycoplasma pneumoniae. (B) Toddlers and young children more frequently develop CAP than middle-aged adults. (C) It is easier to differentiate between typical and atypical pneumonia in pediatric patients. (D) Streptococcus pneumoniae is the most commonly isolated organism in children aged 5 to 15 years. (E) The presence of rhinorrhea, myalgias, or a concomitant illness in a family member is more common in viral pneumonia.

25. Answer B. With the possible exception of elderly patients older than 75 to 80 years, the annual incidence of pneumonia in children under 5 years is higher than at any other time of life. M. pneumoniae is the most common cause of CAP in "school-aged children," 5- to 15-year olds as well as in young adults. Increasingly, Chlamydia pneumoniae is thought to be a common cause of pneumonia in children aged 5 to 15 years, although Mycoplasma remains the chief cause of pneumonia in this group. The most common causes of pneumonia in the neonate, from birth to 3 weeks, are group B Streptococcus, gram-negative enterobacteria (e.g., E. coli), and Listeria monocytogenes. Such infections are uncommon but can be severe when present. Between the ages of 3 weeks and 3 months, Chlamydia trachomatis is most common (i.e., not C. pneumoniae) followed by S. pneumoniae. Between 4 months and 4 years, viruses (e.g., respiratory syncytial virus (RSV), parainfluenza virus, influenza, adenovirus, rhinovirus) are the most common cause. S. pneumoniae remains an important cause of bacterial pneumonia in this age group, but the use of the pneumococcal conjugate vaccine has reduced its role as well as the morbidity associated with CAP caused by S. pneumoniae. It is no easier to differentiate between typical and atypical pneumonia in a pediatric population than in adults. Studies have demonstrated that the presence of symptoms that may suggest a viral etiology to pneumonia such as rhinorrhea, myalgias, or an illness in a family member do not help to determine the cause of pneumonia.

26. Upon starting your shift, you receive sign-out on a 46-year-old male diabetic patient who is being treated with an intravenous insulin drip for diabetic ketoacidosis (DKA). Two hours later, a repeat chemistry panel reveals the following: Na+ 141 mEq/L, Cl− 112 mEq/L, HCO3− 17 mEq/L, blood urea nitrogen (BUN) 16 mg/dL, creatinine 0.9 mg/dL, glucose 278 mg/dL. Which of the following is true? (A) The patient has a mixed high anion gap (AG) ketoacidosis and nonanion gap hyperchloremic metabolic acidosis (HCMA). (B) He should be given subcutaneous insulin and his insulin infusion can be discontinued after 30 to 60 minutes. (C) His DKA is not yet resolved and he requires an ongoing intravenous insulin infusion. (D) The patient should be given bicarbonate replacement therapy. (E) A repeat arterial blood gas (ABG) should be performed to guide further therapy.

26. Answer B. This patient's AG is 12, which is normal. Assuming that the patient has a normal albumin concentration (the major contributor to the AG in healthy patients), the normal AG reflects a resolution of the ketoacidosis. If substantial ketoacids were still present, the AG would be persistently elevated and the patient would require an ongoing insulin infusion. The persistently low bicarbonate signifies that a metabolic acidosis is still present, but it is an HCMA (nonanion gap). The development of an HCMA is a common complication of DKA therapy. This is partly due to the infusion of a large volume of saline, which contains chloride in concentrations far greater than plasma (154 mEq per L). Another major contributor is the loss of ketoanions in the urine that would normally serve as precursors for bicarbonate regeneration. The development of HCMA is benign and requires no therapy. In the setting of normal renal function, it will resolve spontaneously over the next 24 hours due to increased renal acid secretion. Subcutaneous insulin should always be overlapped with an insulin infusion when discontinuing continuous insulin therapy.

27. Which of the following best summarizes the role of chest x-rays in the diagnosis of community-acquired pneumonia (CAP)? (A) Validated clinical rules exist to determine which patients with respiratory symptoms require a chest x-ray. (B) The chest x-ray is the most important test in the diagnosis of pneumonia. (C) Chest x-rays are useful to differentiate between "typical" and "atypical" pneumonia. (D) Lobar consolidation is the most common radiographi

27. Answer B. There are no validated criteria to determine which patients with respiratory symptoms should receive a chest x-ray. Furthermore, while the presence of a lobar infiltrate on a chest x-ray is a classic finding in "typical" bacterial pneumonia, it may also be present in cases of "atypical" pneumonia. Conversely, in the setting of a patient with clinical symptoms of pneumonia, a normal chest x-ray does not preclude a diagnosis of "typical" bacterial pneumonia. Therefore, chest x-rays are not useful to differentiate between typical and atypical pneumonia. However, abnormal findings on a chest x-ray remain central to making a diagnosis of pneumonia in patients with symptoms consistent with pneumonia. Most guidelines (e.g., IDSA, BTS) in developed nations include the presence of an infiltrate on a chest x-ray as central to the diagnosis of pneumonia. Lobar consolidation, however, is present in a minority of chest x-rays in patients with CAP. Chest x-rays that are performed very early in the disease course or in patients who are very dehydrated may be falsely negative, but most x-rays have at least some abnormality. Chest x-rays do not have a role in determining empiric antibiotic therapy in CAP.

28. Which of the following routes of administration causes the fastest onset of action of cocaine? (A) Intranasal (B) Sublingual (C) Oral (D) Inhalation (E) Transdermal

28. Answer D. Cocaine may be taken in a variety of routes, most commonly intranasal and inhalational (crack). Inhalational and intravenous use causes the quickest onset of action, followed by intranasal, and then oral. The duration of action is longest in oral, followed by intranasal, then intravenous/inhalational. The transdermal route is not used for cocaine abuse.

29. A 65-year-old woman pedestrian presents after being struck by a car moving at about 20 mph. She has an obvious, open, deformed leg fracture and was unable to walk at the scene. Her prehospital vital signs are P 105, 85/55, and 100% RA. She is awake and alert and in significant pain. You confirm that her airway, breathing, and pulses are intact. On visual inspection, she has an open tibial shaft fracture and has decreased sensation distal to the fracture. Which of the following is the most important next step in management? (A) Splint application to leg (B) Irrigation of leg wound (C) IV gentamicin and cefazolin (D) Tetanus immunization (E) Chest and pelvis x-rays

29. Answer E. Although the patient has an obvious leg fracture, her hypotension suggests that there may be another, more significant injury present. In general, only a few sites of injury in adults can cause enough hemorrhage to result in hypotension. These include thorax, intraperitoneum, retroperitoneum, pelvis, and bilateral femurs. Tibial fractures by themselves do not usually result in hypotension due to hemorrhage. Appropriate adjuncts to the primary survey include radiographs of the chest and pelvis and focused assessment of sonography in trauma (FAST) scan. These tests should be done before any management of the open tibial fracture. Specific management of the open tibial fracture can occur only if there are extra care providers to perform this concomitantly with the adjuncts to the primary survey. Tetanus immunization can be carried out at any time within 72 hours of the injury and need not be an emergency procedure.

A 62-year-old man presents after a motor vehicle collision with severe ankle pain after his left foot got caught under the accelerator. His foot is shown in Figure 1-1. Which of the following is likely to be true regarding this patient? (A) Ankle sprain is the most likely diagnosis. (B) Accompanying bony fracture is extremely likely. (C) Sciatic nerve injury is likely to be involved. (D) Nonemergent reduction is the most appropriate treatment option. (E) Pain control is rarely needed due to associated nerve injury.

3. Answer B. The appearance of the foot and ankle is highly indicative of an ankle dislocation. The skin between the foot and the ankle appears tented and the foot is rotated relative to the rest of the leg. Ankle dislocations almost always involve a fracture of one or more of the bones in the mortise (malleoli, talus). With this degree of deformity, simple ankle sprain is highly unlikely. Nerves injured in ankle dislocation include the tibial, superficial peroneal, and sural. Emergent closed reduction with splinting is the most appropriate initial management. If there is no neurovascular compromise, it is reasonable to wait for plain radiographs to perform closed reduction. Definitive care usually requires operative management. Pain control is absolutely indicated as total anesthesia with ankle dislocations is rare.

32. An otherwise healthy 12-year-old male is brought to the ED with a chief complaint of chest pain. The patient has been complaining of the pain over the past week. Which of the following is true? (A) The friction rub of pericarditis is best heard when patients are lying supine. (B) Cardiac causes of chest pain more commonly occur in the setting of exertion. (C) In pediatric patients, myocardial infarction is most common in females. (D) On physical examination, only diastolic murmurs are cause for concern of an underlying cardiac problem. (E) Valve disorders are the most common cardiac causes of pediatric chest pain.

32. Answer B. Chest pain is a frequent complaint among pediatric patients presenting to EDs. Fewer than 10% of patients are eventually diagnosed with a cardiac cause of chest pain. Most cases are caused by musculoskeletal etiologies (e.g., costochondritis) or are considered idiopathic. Among cardiac causes, pericarditis and arrhythmias are the most common. Valve disorders are uncommon and, when present, are more often diagnosed in early childhood. Recurrent chest pain that occurs with exertion, chest pain occurring prior to syncope, or a family history of sudden cardiac death each increases the likelihood of an underlying cardiac problem. Pericarditis is a common cardiac cause of pediatric chest pain. Patients with pericarditis typically feel better lying forward and worse when lying supine. However, the friction rub is best heard with the patient leaning forward or even sitting on all fours. These positions bring the heart closer to the chest wall allowing for better auscultation. Myocardial infarction is rare in pediatric patients, but when it occurs, it is most common in males, and it is associated with smoking and substance abuse (primarily cocaine).

34. Which of the following is true regarding posterior shoulder dislocations? (A) External rotation is usually intact. (B) Neurovascular injury is more common than in anterior dislocations. (C) The absence of pain excludes the diagnosis. (D) Seizures are a common mechanism of injury. (E) Recurrent injury is more common than in anterior dislocations.

34. Answer D. Posterior shoulder dislocations occur much less commonly than anterior dislocations. Mechanisms include seizure (due to stronger internal rotator muscles compared with external rotator muscles), electrocution, and fall on an outstretched hand. Patients with posterior shoulder dislocation are almost never able to abduct or externally rotate their affected arms. Neurovascular injury is much less common than with anterior dislocations due to the anterior position of the neurovascular structures. Posterior shoulder dislocation is often confused with adhesive capsulitis and may simply present as stiffness and limited range of motion rather than frank pain. Recurrent injury does occur, but less commonly than in anterior shoulder dislocations. Management is with early reduction and orthopedic consultation.

35. Which of the following is the most effective therapy for acute arsenic poisoning? (A) Activated charcoal (B) Ipecac (C) Dimercaprol (D) Penicillamine (E) Deferoxamine

35. Answer C. Acute arsenic poisoning affects multiple organs, including the liver, kidneys, lungs, and heart. It replaces phosphate in high-energy adenosine triphosphate (ATP) bonds and decreases energy production. Management is supportive plus chelation therapy. The first-line therapy for chelation is dimercaprol (or British antilewisite [BAL]). Activated charcoal does not absorb arsenic or heavy metals. Ipecac is almost never indicated for any poisoning. Penicillamine is a less effective alternative to dimercaprol and is only used when the latter's GI side effects are prohibitive. Deferoxamine is used for chelation of iron.

36. A 42-year-old woman presented to the emergency department (ED) with acute-onset epigastric abdominal pain and nausea without vomiting. Her workup revealed acute pancreatitis and she was admitted. After receiving appropriate analgesics and antiemetics, she is now hungry and wants something to eat. Which of the following is true? (A) She should undergo a period of bowel rest for 48 hours regardless of her laboratory results. (B) She requires ongoing nasogastric (NG) suctioning until pancreatic enzyme abnormalities resolve. (C) She should receive total parenteral nutrition for 72 hours. (D) She should be allowed to eat if abdominal computed tomography (CT) reveals resolution of the signs of pancreatitis. (E) She may eat a low calorie, carbohydrate-rich diet.

36. Answer E. The decision about when to resume feeding and what patients should be allowed to eat is a matter of ongoing controversy. In the past, all patients underwent continuous NG suctioning or were kept NPO. Currently, the only indication for NG suctioning is intractable vomiting or ileus. Some studies suggest that early enteral nutrition may actually improve outcomes. Laboratory and radiographic evidence of pancreatitis is likely to persist until patient discharge, so these are not useful guides for resuming feeding. Many authors now recommend that enteral feeds should be started as soon as a patient is able to tolerate them. Although there is a dearth of evidence regarding the composition of feeds, it is known that pancreatic secretions decrease as carbohydrate composition exceeds 50% of the caloric content of the diet. Therefore, it makes sense to start with a low calorie, carbohydrate-rich diet, and steadily increase both the caloric and the fat content of the diet over a period of days.

38. A 56-year-old woman presents with acute organophosphate overdose, with severe bronchorrhea, bradycardia, and coma. She is intubated for airway protection, and atropine therapy is initiated. After 10 mg of atropine, her heart rate (HR) is 130, blood pressure (BP) is 160/90, and her secretions are still copious. Which of the following is the most appropriate next step in management? (A) Stop atropine, start epinephrine (B) Stop atropine, start vasopressin (C) Stop atropine, start pralidoxime (D) Continue atropine therapy alone (E) Continue atropine therapy and add pralidoxime

38. Answer E. Organophosphates bind to and inhibit acetylcholinesterase, causing a cholinergic syndrome of systemic hypersecretion: bronchorrhea, diarrhea, lacrimation, salivation, emesis, and incontinence. Effects on HR are variable. Mortality from organophosphate overdose is usually due to hypoxia from excessive bronchorrhea. Treatment involves high-dose atropine—the endpoint of atropine therapy is the reduction of bronchial secretions. Tachycardia and hypertension are not indications to stop atropine therapy. Over time, the binding of organophosphates to acetylcholinesterase becomes irreversible. Pralidoxime acts to break up this complex before this process (known as aging) occurs.

39. A 9-month-old infant is brought to the emergency department (ED) with a bruise on his thigh suffered from falling out of his high chair. Radiographs reveal a midshaft femur fracture. Which of the following is the most likely contributing factor? (A) Child abuse (B) Osteogenesis imperfecta (C) Bone tumor (D) Bone cyst (E) Hypocalcemia

39. Answer A. Fractures are found in most abused children. Although no fracture is 100% specific for child abuse, several types are extremely high risk—any fractures in infants (especially of the femur) and spiral, multiple, rib, metaphyseal, humerus, and scapula fractures. Complete skeletal surveys are indicated for patients younger than 5 years who are suspected of being abused. Osteogenesis imperfecta is a rare disease, which causes problems in bone synthesis due to collagen defects. Frequent fractures are common and physical examination may demonstrate blue sclerae, deafness, and ligamentous laxity. Subclinical cases may be more common than previously recognized. Bone tumors and cysts and hypocalcemia may predispose to fracture but are not as likely to cause femoral fractures in the infant as child abuse.

43. Which of the following is true regarding pediatric EKG analysis? (A) Left axis deviation is normal in healthy neonates. (B) Atrial fibrillation is the most common pediatric dysrhythmia. (C) T-wave inversion in the anterior precordial leads (V1-V3) is a normal finding in school-aged children. (D) Cardiac dysrhythmias are the most common electrocardiographic manifestation of underlying congenital heart disease. (E) ST elevation is most commonly associated with myocardial injury.

43. Answer C. T-wave examination has limited utility in pediatric EKG analysis. Upright T waves in the anterior precordial leads (V1-V3) are normal in the neonate, but they invert after the first week of life and remain inverted until early adolescence, at which time they take on the typical adult, upright appearance. In some patients, the "juvenile pattern" of T-wave inversion can persist into a patient's 20s. In contrast, upright T waves in the anterior precordial leads of an otherwise healthy child can be a sign of right ventricular hypertrophy (RVH), which may reflect underlying congenital heart disease. Right axis deviation is a normal finding in healthy neonates due to the large right ventricular mass, but this resolves over time. Left axis deviation is not normally seen. Supraventricular tachycardias are the most common dysrhythmias in pediatric patients. Atrial fibrillation and atrial flutter are rare and are typically only seen in postsurgical patients after congenital heart disease repair. In general, rhythm disturbances are uncommon presenting manifestations of congenital heart disease. Ventricular hypertrophy and axis deviation are common indications of underlying structural heart disease. ST elevation in otherwise healthy children and adolescents is usually due to BER. However, as in adults, ST elevation can indicate myocardial infarction. Since myocardial infarction is exceedingly rare in the pediatric population, other findings supporting a diagnosis of infarction should be sought, such as cardiac enzymes.

44. A 65-year-old smoker presents with sudden onset of shortness of breath for 2 hours. He denies chest pain, fever, productive cough, or lower extremity edema. Vitals signs are 99°F, 95, 24, 200/95, 92% on RA. He has bilateral wheezes. Which of the following best differentiates between obstructive lung disease and congestive heart failure (CHF)? (A) Clinical response to albuterol (B) D-dimer (C) B-type natriuretic peptide (BNP) (D) Troponin I (E) Angiotensin-converting enzyme

44. Answer C. The patient presents with symptoms of either obstructive lung disease or flash pulmonary edema with reactive bronchospasm, also called cardiac asthma. The two entities can be clinically indistinguishable. BNP is released in response to ventricular stretch during CHF. Normal BNP levels significantly reduce the likelihood of CHF exacerbation. Both obstructive lung disease and cardiac asthma may respond to albuterol therapy. D-dimer has a high negative predictive value for PE but has no role in the diagnosis of CHF or obstructive lung disease. Troponin I has high specificity for acute myocardial infarction and would not be elevated in CHF exacerbations without infarction. Angiotensin-converting enzyme levels are used to evaluate sarcoidosis.

46. A 23-year-old man presents to the ED with loss of consciousness after being kicked in the head. He is now awake and alert and complains of a headache. Brain computed tomography (CT) is performed and a slice is shown (Fig. 1-6). Right after the CT scan, he becomes unresponsive. Which of the following is the most appropriate next step in management? (A) Burr hole placement (B) Endotracheal intubation (C) Diagnostic peritoneal lavage (DPL) (D) Emergent thoracotomy (E) Repeat CT scan

46. Answer B. The CT scan shows a right-sided epidural hematoma. While the patient is initially awake and alert, this may be indicative of the lucid interval that is often seen in patients with epidural hematoma (although it is neither sensitive nor specific for the diagnosis). Acute worsening of the clinical status mandates returning to the ABCs of trauma evaluation. Although burr hole placement is the most important definitive management, control of the airway is the most important initial action. CT and FAST scans have largely replaced DPL in the ED. In an unstable patient with a suspected intraperitoneal source of bleeding despite a negative FAST scan, DPL may be performed if the patient is too unstable for CT scan (e.g., persistent, severe hypotension). However, in solitary head trauma, such a scenario is unlikely. Thoracotomy is indicated in patients with penetrating trauma to the chest who arrest in the ED or shortly beforehand. Nasogastric tube placement is contraindicated in patients with severe head and facial trauma as damage to the inferior portion of the skull may allow the tube to pass from the nose into the cranium. (Figure courtesy of Robert Hendrickson, MD. Reprinted from Hendrickson R. Greenberg's Text-Atlas of Emergency Medicine. Lippincott Williams & Wilkins; 2004:51, with permission.)

In setting of a normal peripheral white blood cell (WBC) count and a suspected "traumatic" lumbar puncture, the cerebrospinal fluid (CSF) should contain approximately 1 WBC per every _____ red blood cells (RBCs)? (A) 1 (B) 50 (C) 250 (D) 500 (E) 1,000

47. Answer D. An often used rule of thumb is to subtract 1 WBC for every 1,000 RBCs present in the CSF among patients with a normal peripheral WBC. However, to get a more accurate estimate of the actual appropriate number of WBCs in the CSF, the following formula should be used: ([CSF RBC] × [peripheral blood WBC]/[peripheral blood RBC] = predicted CSF WBC).

48. Which of the following is true regarding acute mesenteric ischemia (AMI)? (A) Most patients have a lactic acidosis early in the course of their illness. (B) The mortality rate of AMI is roughly 70%. (C) In the absence of angiography, intravenous heparin infusion is the standard of management. (D) Tenderness on physical examination is most often worse than a patient's subjective complaint of pain. (E) The most common CT finding is gas in the portal venous system.

48. Answer B. Owing to its deceptively innocuous early course, the diagnosis of AMI remains problematic. Therefore, the mortality rate has remained essentially unchanged at roughly 70%. The key to diagnosis is recognizing patients at risk, such as any patient older than 50 years of age who presents with acute abdominal pain and who has known vascular disease, cardiac arrhythmias, recent myocardial infarction, hypovolemia, hypotension, or sepsis. The most commonly cited clinical finding is pain that is out of proportion to tenderness elicited on physical examination. This is a nonspecific finding that needs to be considered carefully in light of the clinical scenario. Unfortunately, there are no laboratory markers or radiologic studies apart from angiography that have sufficient sensitivity and specificity to exclude AMI early in its course. Lactate levels are elevated in approximately 100% of patients with bowel infarction, but this is a late finding and mortality rates are high by the time infarction has occurred. Plain films are most commonly nonspecific, although findings such as ileus correspond to more severe disease and a higher mortality rate. The sensitivity of CT has been cited to be as high as 82%, but the most common early finding is bowel wall thickening, present in 26% to 96% of cases. Unfortunately, this is also the least specific finding and is often not present in mesenteric ischemia due to arterial embolism or thrombosis, which is the most common cause of AMI. Pneumatosis intestinalis or gas in the portal venous system is a specific finding but is only present after bowel infarction has occurred. In the absence of angiography, the treatment is emergent laparotomy.

50. A 27-year-old woman at 33 weeks of gestation presenting with liver tenderness and evidence of coagulopathy is most suggestive of: (A) Acute fatty liver of pregnancy (AFLP) (B) Preeclampsia (C) Hepatitis (D) Cholecystitis (E) Intrahepatic cholestasis of pregnancy (ICP)

50. Answer B. The HELLP syndrome is a severe manifestation of preeclampsia characterized by hemolysis, elevated liver enzymes, and low platelets. However, liver function tests are abnormal in 20% to 30% of patients overall and is not limited to patients with HELLP syndrome. AFLP is an exceedingly rare disorder (at least 10 times less common than HELLP syndrome) that occurs during the third trimester. It is heralded by the presence of nausea and vomiting in the third trimester with associated epigastric pain and liver dysfunction. In contrast to the HELLP syndrome, coagulation abnormalities, including elevated PT, are present early in the disease course. Hepatitis is the most common cause of liver disease in pregnancy. As in nonpregnant women, however, most patients experience a subclinical illness and do not report any symptoms. Symptomatic patients present with jaundice or scleral icterus, nausea and vomiting, and right upper quadrant tenderness with aminotransferase levels in the thousands. The course in pregnant and nonpregnant women is typically benign and indistinguishable from one another. Cholecystitis is the second most common surgical emergency during pregnancy, and patients typically present with fever, right upper quadrant pain, nausea and vomiting, and leukocytosis (although this can be confused with the leukocytosis of pregnancy). ICP is, like AFLP, a rare disorder that typically complicates the third trimester. Patients present with moderate to severe pruritus that typically begins on the palms and soles and progresses in an ascending manner. Approximately 20% of patients will also be jaundiced on presentation. ICP is associated with increased preterm delivery, increased perinatal mortality, and meconium staining. Fetal mortality approaches 20% in untreated patients. The optimal treatment for preeclampsia, AFLP, and ICP is delivery.

51. Which of the following results from a lesion within the pons? (A) Ipsilateral oculomotor nerve palsy with contralateral hemiparesis. (B) Upward gaze paralysis with torsional nystagmus. (C) Ipsilateral facial droop with contralateral hemiparesis. (D) Ipsilateral lateral gaze palsy with ipsilateral hemiparesis. (E) Deviation of the tongue away from the lesion and contralateral hemiparesis.

51. Answer C. The hallmark of lesions within the brainstem is the presence of "crossed signs." This refers to the occurrence of ipsilateral brainstem lesions with contralateral hemiparesis and hemisensory loss. The oculomotor nucleus is located in the midbrain, whereas the hypoglossal nucleus is located in the medulla. Therefore, neither of these nerves is affected by pontine lesions. Although the abducens nucleus is located within the pons, a pontine lesion resulting in ipsilateral lateral gaze palsy should be accompanied by contralateral hemiparesis. Of note, because brainstem lesions affect cranial nerve nuclei (or lower motor neurons of cranial nerves), such lesions result in a complete deficit in the distribution of the nerve. With respect to the facial nerve, this results in complete facial droop rather than the forehead sparing seen with cortical lesions (due to bilateral innervation of the facial nucleus).

53. A 34-year-old man presents after a high-speed motor vehicle crash. Chest x-ray is performed and shown in Figure 1-7. Which of the following is the most likely diagnosis? (A) Pneumothorax (B) Small bowel rupture (C) Duodenal hematoma (D) Diaphragmatic rupture (E) Hemothorax

53. Answer D. The chest x-ray demonstrates presence of the gastric bubble inside the thoracic cavity, indicating a large diaphragmatic rupture from blunt trauma. The left diaphragm is much more likely than the right to rupture due to protection on the right from the liver. Management involves nasogastric decompression and surgical repair. Patients with penetrating thoracoabdominal trauma may have delayed abdominal herniation into the thorax as far out as decades after the initial injury. Rightsided diaphragmatic injuries are less likely to have abdominal herniation into the thorax, also because of the presence of the liver. Diaphragmatic injuries may be very subtle and diagnosed only on direct visualization with laparoscopy or thoracoscopy. DPL, FAST, and CT scan all lack sufficient sensitivity for ruling out the diagnosis. (Figure courtesy of Mark Silverberg, MD. Reprinted with permission from Silverberg M. Greenberg's text-atlas of emergency medicine. Lippincott Williams & Wilkins; 2004:654.)

54. A 15-year-old previously healthy female is brought to the ED by her parents with a chief complaint of SOB, palpitations, lightheadedness, and severe fatigue. The patient had been diagnosed with "a viral infection" by her pediatrician five days before presenting to the ED. While she initially seemed to improve, she complains of feeling SOB with a sense that her heart is "beating out of her chest" for the past day. Her parents noted she was breathing fast, and she appeared to be "winded" even when sitting down. Upon arrival, she is pale and ill-appearing, with vitals of T 98.6, P 143, RR 28, BP 105/70, and SaO2 of 92% on RA. A chest x-ray demonstrates fluffy bilateral infiltrates. Her lung examination reveals tachypnea with increased work of breathing, and rales without wheezes. Which of the following is true of the patient's likely underlying problem? (A) The patient's EKG will most likely reveal supraventricular tachycardia. (B) An elevated MB isoenzyme of creatine kinase (CK-MB) helps to confirm the patient's diagnosis and direct therapy. (C) Loop diuretics and inotropes may be helpful in her management. (D) Two set of blood cultures should be drawn and IV antibiotics, and aggressive IV fluids should be administered to treat a likely superimposed bacterial infection. (E) Echocardiography will likely reveal previously undiagnosed congenital heart disease.

54. Answer C. This patient is presenting with acute CHF, likely due to virally mediated myocarditis. While patients with acute myocarditis often present in a more subtle manner than this patient, patients may present with a broad range of symptoms, from vague generalized malaise to fulminant heart failure. Therefore, the ED physician must maintain a high degree of suspicion in patients presenting to the ED with new symptoms in the convalescent phase of a viral illness. The most common arrhythmia in patients with myocarditis is sinus tachycardia. Patients rarely present with supraventricular tachycardia, but when present, the rate is typically 180 to 260 beats per minute. Though the CK-MB and troponin are frequently measured in adults with chest pain, CK-MB has not been shown to be either sensitive or specific for pediatric patients with myocarditis. Elevated troponin levels also demonstrate poor sensitivity (34%) but relatively high specificity (82%) for myocarditis. Therapy is not based on the levels of any biomarker. Since this patient is presenting with fulminant heart failure, loop diuretics (e.g., furosemide) and inotropes (e.g., dobutamine, milrinone) may be essential in her management. Vasodilators (e.g., nitroglycerin) are also a mainstay of therapy, but this patient's relative hypotension may limit their use. Early consultation with a pediatric cardiologist is also essential as the patient should be transferred to a center with the capability of providing extracorporeal membrane oxygenation, should that become necessary. The patient's lack of fever or cough points against pneumonia as a likely diagnosis. Echocardiography is an essential diagnostic tool, but patients only rarely have preexisting, undiagnosed congenital heart disease. Most often, echocardiography reveals global left ventricular dysfunction, with variable degrees of left ventricular cavity enlargement.

55. A 55-year-old woman with chronic obstructive pulmonary disease (COPD) presents with acute dyspnea, purulent cough, and fever. Vital signs are 100.4°F, 110, 22, 175/90, 98% on 4L NC. She has mild neck muscle retractions and bilateral rhonchi. Chest x-ray shows no focal infiltrate. Albuterol nebulizer, methylprednisolone 125 mg IV, and levofloxacin 500 mg IV have only minimally improved respiratory distress. Which of the following is the next best step in management? (A) Azithromycin 500 mg IV (B) Vancomycin 1 g IV (C) Aminophylline 5 mg per kg IV (D) Noninvasive positive pressure ventilation (E) Endotracheal intubation and mechanical ventilation

55. Answer D. The patient is not responding to initial therapy for COPD exacerbation. She is in moderate respiratory distress but has a normal mental status. In patients with COPD, noninvasive positive pressure ventilation is an excellent alternative to endotracheal intubation and mechanical ventilation. It reduces the risk of endotracheal intubation, ICU length of stay, and overall hospital length of stay. Azithromycin adds little to fluoroquinolone coverage for common pathogens in COPD exacerbations. Vancomycin is used to treat methicillin-resistant S. aureus, but this would not be suspected in a patient without frank pneumonia or nosocomial exposure. Aminophylline is a phosphodiesterase inhibitor that is used in asthma exacerbations but is not as effective in COPD exacerbations.

56. Which of the following is true regarding electrical injury? (A) Direct current (DC) is more dangerous than alternating current (AC). (B) In high-voltage injuries, the extent of cutaneous burns is a good predictor of internal tissue damage. (C) Asystole is the most common dysrhythmia resulting from low-voltage electrical injury. (D) In contrast to other mass casualty traumatic events, patients without signs of life should be resuscitated first. (E) All of the above.

56. Answer D. For a given voltage, AC is thought to be three times more dangerous than DC. This is due to the fact that AC current causes repetitive muscle contraction or tetany once the "let-go current" is exceeded. This results in prolonged exposure and more severe injury. High-voltage electrical injuries should be treated like crush injuries, because there is often a large amount of tissue damage underneath normal appearing skin. It is impossible to predict the degree of underlying damage from the extent of cutaneous burns. Fewer than 10% of patients experiencing low-voltage electrical injury develop a cardiac dysrhythmia. In those patients who do suffer cardiac arrest due to an arrhythmia, ventricular fibrillation is most common. Triage priorities are different in cases of high-voltage electrical injury or lightning strikes. Patients with obvious signs of life tend to do well and can afford a small delay in definitive care. Furthermore, due to the possibility of a good outcome with cardiopulmonary resuscitation (CPR), patients without signs of life should receive immediate care.

57. Which of the following is true regarding cluster headaches? (A) The pain is typically bilateral. (B) They occur more commonly in men. (C) The average age of onset is 45 years. (D) The average cluster period lasts for 1 week. (E) Pain typically occurs in the V2 distribution of the trigeminal nerve.

57. Answer B. Of the primary headache syndromes, cluster headaches have the most consistent presentation. The average age of onset is 28 to 30 years. They are strictly unilateral headaches that occur in the ophthalmic division (V1) of the trigeminal nerve. Therefore, the pain is most commonly maximal in the retroorbital and temporal region, although patients may experience radiating pain to the forehead, temple, cheek, and jaw. The pain is severe and is often described as "boring" or "tearing" in nature. Patients are often very restless or agitated during cluster headaches and characteristically rock or pace back and forth. The headaches last on average from 45 to 90 minutes and occur one to three times daily. The typical cluster period (during which headaches occur) lasts 6 to 12 weeks with typical remissions lasting 12 months. Traditionally, they have been much more common in men, although recent epidemiologic studies have revealed a declining predominance of men. In addition, patients affected by cluster headaches more frequently indulge in cigarette smoking and alcohol abuse. Interestingly, once a cluster period begins, alcohol usually triggers an attack within minutes. There is also a seasonal pattern to the clusters, with exacerbations occurring more often in the spring and fall.

58. A 6-year-old girl is brought to the emergency room 4 hours after developing a brief choking episode while playing with her toys. Her chest x-ray is shown in Figure 1-8. Where is the foreign body located? (A) Esophagus (B) Hypopharynx (C) Trachea (D) Anterior mediastinum (E) Not possible to determine from the information provided

58. Answer A. Diagnosis of aspirated foreign bodies relies on plain radiographs. Posteroanterior/lateral chest x-rays and AP and lateral soft-tissue neck films are diagnostic in the case of radiopaque esophageal and tracheal foreign bodies. Esophageal foreign bodies are seen "en face" in AP views and on edge in lateral views, whereas the opposite is true for tracheal foreign bodies. Frequently, both the AP and the lateral views will be needed to determine the exact location of the foreign body. Additionally, the patient may present with ongoing symptoms that provide further clues to the location of the foreign body. Such symptoms include dysphagia, odynophagia, or regurgitation of food in the setting of esophageal obstruction, or stridor, wheezing, or generalized respiratory distress in the case of tracheal foreign bodies. (Figure reprinted with permission from Fleisher GR. Atlas of Pediatric Emergency Medicine. Lippincott Williams & Wilkins; 2003.)

59. A 32-year-old pregnant woman in her third trimester presents with dysuria. She has a penicillin allergy. Urinalysis demonstrates bacteria, but no white blood cells, leukocyte esterase, or nitrites. Physical examination is normal. Which of the following is the most appropriate next step in management? (A) Close outpatient observation (B) Trimethoprim-sulfamethoxazole (C) Ciprofloxacin (D) Nitrofurantoin (E) Amoxicillin

59. Answer D. Both bacteriuria and urinary tract infection (UTI) should be treated aggressively in pregnant women, as they can cause serious complications with delivery. The prevalence of bacteriuria may be higher in pregnant women than in nonpregnant women, but UTI rates are comparable. Symptoms are identical between the two groups. Gram-negative enteric bacteria cause the vast majority of UTIs in pregnant women, but a crucial organism to consider is Group B Streptococcus, which can cause serious neonatal infection. Nitrofurantoin, penicillins, and cephalosporins are the drugs of choice. Sulfonamides are not safe during the third trimester because of the possibility of neonatal hemolysis. Fluoroquinolones cause various congenital defects. Although amoxicillin would be a reasonable choice in most patients, the allergy to penicillin is an obvious contraindication.

62. Patients with botulism most classically present with which of the following? (A) Ascending symmetric anesthesia. (B) Nausea, vomiting, and lower extremity weakness within 1 to 2 hours of toxin exposure. (C) Descending symmetric paralysis. (D) Anticholinergic symptoms of constipation, dry skin, dry eyes, and urinary retention. (E) Altered mental status.

62. Answer C. Botulinum toxin works by irreversibly binding to the presynaptic membrane of peripheral and cranial nerves where it inhibits the release of acetylcholine. The patient improves as new receptors are manufactured. Because the disorder is localized to the neuromuscular junction, there are no sensory findings. The classic presentation is descending, symmetric paralysis which typically starts in the bulbar muscles. Such patients present with dysarthria, diplopia, and dysphagia which progresses to generalized weakness. The toxin does produce anticholinergic symptoms due to its inhibition of cholinergic output. Therefore, the patient's pupils may be dilated and unreactive, which can be a useful feature for differentiating such patients from those with myasthenia gravis. Gastroenteritis may or may not occur, but the onset of symptoms is not until 6 to 48 hours after ingestion. Mental status remains intact.

63. The pictured deformity in Figure 1-10 occurs in: (A) Rheumatoid arthritis (B) Osteoarthritis (C) Systemic lupus erythematosus (D) Reactive arthritis (E) Psoriatic arthritis

63. Answer A. This is the swan neck deformity. It is caused by hyperextension at the proximal interphalangeal (PIP) joint and flexion at the distal interphalangeal (DIP) joint. The DIP joint flexion occurs due to elongation or rupture of the extensor tendon attachment to the distal phalanx (i.e., similar to a mallet injury). Left untreated, PIP hyperextension occurs as a consequence of the distal mallet deformity. However, the deformity can begin in the PIP joint as well due to synovitis of the volar capsule resulting in PIP hyperextension. In the latter case, DIP flexion occurs as a secondary effect. Both the swan neck and boutonniere deformities are common in rheumatoid arthritis. (Figure reprinted with permission from Oatis CA. Kinesiology: The mechanics and pathomechanics of human movement. Lippincott Williams & Wilkins; 2004.)

64. A 42-year-old man presents with left foot pain. He states his foot folded underneath him after he jumped from a short ledge during a game with his son. His body weight first landed on his plantar-flexed foot and then the foot folded beneath him. He now has difficulty ambulating. An x-ray is shown in Figure 1-11. Which of the following is true? (A) The fracture fragment indicates that the ligaments are intact. (B) This is a stable injury. (C) Degenerative arthritis is the most common complication. (D) Subtle plantar displacement is sometimes the only sign of this injury. (E) The posterior tibial artery may be damaged during this injury.

64. Answer C. The image reveals a Lisfranc fracture-dislocation. Lisfranc injuries are uncommon injuries most often resulting from high-energy trauma such as motor vehicle accidents. They may also occur because of axial loading on a foot in extreme plantarflexion. Any displacement indicates inherent instability and patients develop a high rate of degenerative arthritis, even in the setting of operative repair. Owing to the low incidence of Lisfranc injuries as well as the significant trauma that is often required to produce such an injury, Lisfranc injuries are often overlooked. Furthermore, radiographic evaluation can produce subtle findings. Dorsal displacement of the metatarsals on a lateral foot radiograph is one such finding and indicates ligamentous disruption. Plantar displacement does not occur because of the strength of the plantar fascia. Most commonly, however, radiographs will reveal a misalignment of the second metatarsal and the medial border of the middle cuneiform. In addition, the medial border of the fourth metatarsal should be aligned with the medial border of the cuboid on an oblique view. Any displacement indicates a Lisfranc injury. Trauma to the dorsalis pedis artery may occur with Lisfranc injuries as the dorsalis pedis descends between the first and second metatarsals to form the plantar arch. (Figure reprinted with permission from Harris JH. The radiology of emergency medicine, 4th ed. Lippincott Williams & Wilkins; 1999:885.)

7. Which of the following is true regarding temporomandibular joint (TMJ) syndrome? (A) It is an extremely rare cause of facial pain. (B) Young women are at highest risk. (C) Pain is normally bilateral. (D) Muscle relaxants are not helpful in management. (E) Avoidance of hard foods is rarely necessary.

7. Answer B. TMJ syndrome refers to a vague set of disorders involving the TMJ, such as pain, joint locking, and dislocation. It is considered the most common cause of facial pain after dentalgia. Young women comprise the highest risk category, and many patients have concomitant psychiatric conditions. The pain is normally unilateral. Evaluation involves imaging to assess for structural abnormalities and laboratory tests to check for associated systemic diseases such as RA, degenerative joint disease, and ankylosis. Treatment is with nonsteroidal anti-inflammatory drugs (NSAIDs), muscle relaxants, and a soft food diet during acute episodes to prevent further exercise of the muscles of mastication.

65. A 41-year-old female is referred from her dentist because her blood pressure was 212/105 when she presented for her root canal. She complains of dental pain in the area of the affected tooth, but she denies headache, blurry vision, nausea, or extremity weakness or numbness. The remainder of her review of systems is negative. Her physical examination is unrevealing except for carious teeth. She has never been diagnosed with hypertension, but admits that she had not seen a physician "in years." She has no drug allergies or known contraindications to any specific drug therapy. Which of the following is the next best step? (A) Administer Clonidine 0.1 mg PO every hour until her blood pressure is less than 185/110 and then discharge. (B) Obtain a chest x-ray, EKG, head CT, CBC, basic metabolic panel, and cardiac enzymes to help determine her disposition. (C) Start the patient on Hydrochlorothiazide 25 mg PO daily, and discharge her to follow-up with her primary care physician. (D) Administer one dose of IV hydralazine until the patient's BP is lower than 185/110, then switch to oral therapy, and discharge if asymptomatic. (E) Initiate an IV labetalol infusion and admit the patient with a diagnosis of hypertensive urgency.

65. Answer C. Asymptomatic hypertension is an enormous, often mismanaged, problem in EDs for which there is little data to guide emergency physicians. Most of the confusion arises from the use of the term "hypertensive urgency." Hypertensive urgency has been variably defined but generally has included any patient with a blood pressure greater than 185/110 regardless of symptoms. There is no evidence that the "urgent" treatment of such patients is helpful. In contrast, many patients have been needlessly treated with an overly aggressive approach that has led to more harm than good. Asymptomatic patients require no screening testing as long as appropriate follow-up can be arranged. Physicians may choose to initiate low-dose, mild therapy, such as hydrochlorothiazide or chlorthalidone, as a bridge to more comprehensive therapy. Clonidine is not a first-, second-, or third-line agent for the treatment of hypertension and has no role in the ED management of hypertension unless it is part of the patient's existing regimen. There is no role for admission, extensive testing, or IV therapy in asymptomatic patients. Symptomatic patients should undergo a workup based on their symptoms, and should only receive IV therapy if there is also an emergent condition necessitating an emergent reduction in blood pressure. It is reasonable to check renal function if starting antihypertensive therapy, particularly with ACE inhibitors, but it is not mandatory if the patient has close follow-up.

66. A 45-year-old man presents with abrupt onset of fever, cough, chills, and shortness of breath. One week earlier, he was present in a government office where an explosion thought to be a terrorist attack occurred. He appears toxic and in moderate respiratory distress. Chest x-ray is shown in Figure 1-12. Which of the following is the most appropriate therapy at this time? (A) Ciprofloxacin (B) Aztreonam (C) Tobramycin (D) Ceftriaxone (E) Methylprednisolone

66. Answer A. The patient likely has pulmonary anthrax due to the Gram-positive organism Bacillus anthracis. Anthrax is a CDC Category A agent of bioterrorism, with the highest risk of weaponization and transmission. Pulmonary anthrax is caused by the inhalation of anthrax spores, which causes a pneumonia-like picture a week after exposure. Hemorrhagic mediastinitis is the characteristic radiographic manifestation, though CT scan should be ordered if initial chest x-ray is negative. First-line therapy is with ciprofloxacin and second line is with doxycycline. Aztreonam and tobramycin both cover Gram-negative bacteria with minimal Gram-positive coverage and should not be used. Anthrax has demonstrated intermediate resistance to ceftriaxone. Corticosteroids such as methylprednisolone are not routinely indicated in the treatment of anthrax. Anthrax spores should be disinfected with bleach solution, as ordinary alcohol solutions have no effect. (Figure courtesy of John H. Harris, William H. Harris. Radiology of Emergency Medicine. 4th ed. Lippincott Williams & Wilkins; 1999, with permission.)

67. A 55-year-old woman presents with left knee pain after being struck at 5 miles per hour by a car turning into her pedestrian lane. Her knee is significantly tender inferior and lateral to the patella. She is able to fully extend her leg though with considerable pain and her neurovascular and ligamentous exams are intact. Knee radiographs are normal. Which of the following is the most appropriate next step in management? (A) Discharge with full weight bearing (B) Discharge with crutches and weight bearing as tolerated (C) Computed tomography (CT) scan of the knee (D) Admit for knee observation (E) Urgent arthroscopy

67. Answer C. Patients with inferolateral knee pain and tenderness after trauma are at significant risk for tibial plateau fractures, which are often not visible on plain knee radiographs. Unlike simple knee sprains, management of tibial plateau fractures involves no weight bearing for several weeks. Diagnosis should, therefore, be pursued with advanced imaging techniques, such as CT or MRI. Discharging a patient with potential for tibial plateau fracture with any weight bearing will result in improper healing and increased morbidity. Furthermore, clinicians should be wary of any patient who has signs or symptoms of persistent discomfort after trauma. Observation of knee injuries is not required except in cases of suspected knee dislocation. Urgent arthroscopy is almost never indicated acutely in cases of knee trauma.

68. Which of the following is true regarding temporomandibular joint (TMJ) dislocations? (A) Most of them are posterior. (B) The patient is unable to open the mouth in bilateral dislocations. (C) The jaw is rotated toward the affected side in a unilateral dislocation. (D) The patient will present with a protruding mouth. (E) All of the above

68. Answer D. TMJ dislocations are anterior dislocations that may result after trauma or after widely opening the mouth as in yawning, laughing, or singing. The mandibular condyle slides anteriorly during normal mouth opening. Occasionally the mandibular condyle may slide anterior to the articular eminence of the temporal bone resulting in anterior dislocation. Subsequent muscle spasm prevents spontaneous reduction and the patient presents to the ED with a fixed, protruding, and widely open mouth. The patient may not be able to handle the secretions and may therefore be drooling. Posterior and lateral dislocations can occur but they almost always occur in the setting of trauma. Most spontaneous dislocations are bilateral. When unilateral dislocations occur, the jaw is rotated toward the unaffected side. Reduction is achieved by applying downward pressure on the most posterior molars followed by slight posterior pressure. Gauze should be thickly wrapped around the physician's thumbs to prevent injury from the typical bite that occurs upon reduction.

69. A 4-year-old boy is brought to the ED with a severe sore throat and a history of "refusing to eat." He has severe pharyngitis on examination, but a lateral neck x-ray is taken that you feel is consistent with a retropharyngeal abscess (RPA). You are surprised to find, however, that the patient's subsequent CT was normal. The radiologist tells you this was probably due to poor technique. What technique should be used to most accurately assess the prevertebral space on x-ray? (A) X-ray should be taken in flexion during expiration. (B) The patient should be sitting upright when the x-ray is taken. (C) The x-ray should be taken in flexion during inspiration. (D) The x-ray should be taken in extension during expiration. (E) The x-ray should be taken in extension and inspiration.

69. Answer E. To avoid obtaining x-rays demonstrating a falsely widened retropharyngeal space, lateral neck x-rays should be taken with full extension (cervical lordosis should be visible) and during inspiration. Although there are several criteria or rules, the most common rule is that the prevertebral space should be no wider than the width of the vertebral body behind it. Specifically, the anteroposterior width of the prevertebral soft tissue should be ≤7 mm at the level of C2 (from the anteroinferior border) and <14 mm at the level of C6 in kids, and 22 mm at C6 in adults. At C1-3, one rule of thumb is that the adjacent prevertebral tissue should not be greater in width than half the width of the adjacent vertebral body. Because RPAs are most common in children younger than 4 years old, compliance with extension films during inspiration is difficult to achieve. CT scanning can resolve this difficulty although sedation (which carries its own risks) may be required.

70. Which of the following is the most common cause of acute respiratory distress syndrome (ARDS)? (A) Sepsis (B) Near drowning (C) Multiple blood transfusions (D) Multiple blunt trauma (E) Pancreatitis

70. Answer A. Sepsis is the most common risk factor or condition leading to the development of ARDS. ARDS is defined as the development of acute respiratory failure, with noncardiogenic pulmonary edema (established by the presence of bilateral infiltrates consistent with pulmonary edema on chest radiography and a pulmonary capillary wedge pressure ≤18 mm Hg indicating that the edema is noncardiogenic), and severe hypoxia such that the PaO2:FiO2 ratio ≤200. Identical findings in patients with a PaO2:FiO2 ratio ≤300 are diagnosed with acute lung injury (ALI), instead of ARDS, but the etiology of ALI is the same. Although sepsis is the most common cause of ARDS, severe trauma complicated by shock, multiple blood transfusions, and aspiration are all independent risk factors. Pancreatitis and near drowning may result in ARDS but are less common causes. Such causes also illustrate the idea that the ARDS may be due to direct lung injury (e.g., severe pneumonia with sepsis) or indirect lung injury (e.g., severe pancreatitis). The latter is presumably due to the widespread release of cytokines and other proinflammatory mediators.

71. Which of the following is useful as a sensitive screening test for clinically significant complications of blunt cardiac injury? (A) Creatine kinase, MB isoenzyme (CK-MB) (B) Troponin I (C) Troponin T (D) EKG (E) Exercise stress test

71. Answer D. Blunt cardiac injury results from blunt trauma directed at the sternum, usually from patients striking the steering wheel in a motor vehicle collision. Patients with blunt cardiac injury (formerly known as cardiac contusion) may develop myocardial stunning, CHF, dysrhythmia, and in rare instances, when a coronary vessel is damaged, MI. The diagnosis should be suspected in any case of blunt thoracic trauma, but physical examination is often not revealing. The best screening tool for the diagnosis is EKG. Patients with any significant abnormal finding on EKG should be admitted for observation, telemetry monitoring, and confirmatory echocardiogram. Cardiac markers have been extensively studied to evaluate for screening or confirming the diagnosis but are not particularly useful in either regard. Stress testing is not indicated in patients with suspected blunt cardiac injury, as the tachycardic response may actually exacerbate the traumatic insult.

72. A 46-year-old woman presents to the ED with cough, fever, and dyspnea. Her chest x-ray is shown in Figure 1-13. You initiate empiric treatment for communityacquired pneumonia and perform a diagnostic thoracentesis because of the size of the effusion and her declining respiratory status. Which of the following results on pleural fluid compels you to perform urgent tube thoracostomy? (A) pH <7.0 (B) Malignant cells (C) Protein consistent with exudative effusion (D) Elevated amylase (E) Gram stain negative

72. Answer A. Complicated parapneumonic effusion and empyema are rare, but severe complications of pneumonia are associated with a mortality rate of 15% to 20%. Although diagnostic thoracentesis is a procedure that is rarely performed in the ED, it is necessary to diagnose life-threatening conditions such as empyema as soon as possible. Approximately 60% of patients with pneumonia develop some amount of pleural fluid, but most of these patients have a simple parapneumonic effusion. The characteristics of pleural fluid in the setting of pneumonia determine whether the effusion is simple, complicated, or if frank pus is present, empyema. These three distinct categories of disease represent the typical progression of disease in patients with parapneumonic effusions. Pleural fluid pH, lactate dehydrogenase (LDH), glucose, and Gram stain determine into which category the effusion is classified. Simple parapneumonic effusions are clear, with a pH >7.2, LDH <1,000, glucose >40 mg per mL, and a negative Gram stain. Conversely, complicated parapneumonic effusions may be turbid, have a pH <7.2, LDH >1,000 (reflecting more severe inflammation), glucose <40 mg per mL, and may have a positive Gram stain. A pH <7.0 is typically found only in the setting of frank pus in the pleural space, called empyema. Indications for tube thoracostomy include a pH <7.2 (i.e., a complicated parapneumonic effusion), a positive Gram stain, or the presence of frank pus in the pleural space. Infected parapneumonic effusions must be drained as soon as possible to prevent the development of subsequent adhesive disease resulting in significant loculations and scar tissue. A chest surgeon should be consulted anytime a chest tube is inserted for empyema or complex parapneumonic effusion, because surgery may ultimately be necessary to ensure adequate drainage. (Figure reprinted with permission from Harris JH. The Radiology of Emergency Medicine, 4th ed. Lippincott Williams & Wilkins; 1999:444.)

73. Which of the following is an indication for antibiotic prophylaxis in patients with prosthetic heart valves? (A) Endoscopic retrograde cholangiopancreatography (ERCP) (B) Endotracheal intubation (C) Vaginal delivery (D) Dilation and curettage (E) Local anesthesia

73. Answer A. Endocarditis prophylaxis with antibiotics for patients with prosthetic or damaged valves is recommended for the following procedures: Dental cleaning, rigid bronchoscopy, ERCP, and cystoscopy.

74. Which of the following is an ingredient in sitz baths? (A) Baking soda (B) Salicylate (C) Corticosteroids (D) Insoluble fiber (E) None of the above

74. Answer E. Sitz is a word that comes from the German word, sitzen, meaning "to sit." A sitz bath refers to any device that allows a patient to immerse only the perineum and buttocks in water while draping the rest of their body outside the tub. While patients may add various medications to the water, it is not recommended and there is no evidence that such additives help. Sitz baths are thought to help a variety of perianal complaints, such as hemorrhoids, because anal canal pressure decreases significantly in warm water (40°C) and blood flow improves.

75. Which of the following most places women at risk for abruptio placentae? (A) Preeclampsia (B) Cigarette smoking (C) Premature rupture of membranes (PROM) (D) Cocaine use (E) Trauma

75. Answer A. Increasing maternal age, cigarette smoking, cocaine use, twin or multiple gestation, preexisting or pregnancy induced hypertension (preeclampsia), trauma, chorioamnionitis, oligohydramnios as well as thrombophilias all place women at risk for abruptio placentae. Of these, preeclampsia is the most significant risk factor.

A 63-year-old male with a history of nephrolithiasis presents to the ED with acute onset right flank pain. He receives an uncontrasted CT scan which is unrevealing, and he has no hematuria. A repeat study with contrast is performed, a slice of which is shown in Figure 1-14. The patient feels better and has minimal pain. His vital signs are T 98.4, P 84, RR 16, BP 156/82, SaO2 98% on RA. Which of the following is true? (A) His blood pressure is already at the treatment goal. (B) More than 90% patients have a diagnosis of hypertension. (C) EKGs are helpful in discriminating this from myocardial infarction. (D) Elevated D-dimer levels increase the likelihood of the diagnosis. (E) An upper extremity pulse deficit is the most specific physical examination finding.

76. Answer E. Acute aortic dissection is a hypertensive emergency (see Fig. 1-18). Initial treatment should focus on reducing the patient's blood pressure to a systolic blood pressure of 100 to 120 mm Hg using parenteral beta-blockers (labetalol, esmolol, propranolol). Beta-blockers are the preferred agent because they also reduce heart rate and aortic wall stress by reducing the rate of systolic blood pressure rise. While hypertension is the most common risk factor, approximately 25% of patients have no history of hypertension and approximately one-third are normotensive at presentation. EKGs are not usually helpful to differentiate acute aortic dissection from myocardial infarction as most EKGs in patients with either condition are nonspecific. There are ongoing investigations into the use of low d-dimer levels as a means of excluding aortic dissection. However, there are no validated prediction rules to determine the exact pretest probability in patients with suspected aortic dissection, and no outcome studies have demonstrated the safety of using "negative" d-dimer levels as a means to rule out aortic dissection. As with pulmonary embolism, elevated d-dimer levels do not significantly increase the likelihood of acute aortic dissection. An upper extremity blood pressure disparity is a classically described, though somewhat unreliable finding in patients with aortic dissection. A pulse deficit between the upper extremities is the most reliable physical examination finding, and pulses should always be assessed to determine whether there are discrepancies. Unfortunately, the sensitivity of this finding is low, as only 15% of patients in the International Registry of Acute Aortic Dissection (IRAD) study had such a deficit.

78. Which of the following is true regarding treatment of otitis media? (A) Few cases will resolve spontaneously without antibiotics. (B) Oral antibiotics are superior to intramuscular (IM) antibiotics in efficacy of treatment. (C) High-dose amoxicillin (80 mg/kg/day) should be reserved for those patients who are older than 2 years of age. (D) Otitis media represents the number one reason for outpatient antimicrobial prescriptions in the United States. (E) Auralgan may be beneficial in patients with tympanic membrane perforations.

78. Answer D. Otitis media is the number one reason for antibiotic prescriptions in the United States, despite the fact that more than 75% of cases will resolve spontaneously without treatment. The 2004 American Academy of Pediatrics (AAP) and American Academy of Family Physicians (AAFP) joint guidelines recommend treatment with antibiotics for patients younger than 2 years old, and consider an observation period for patients older than 2 years old an appropriate management strategy if the patient does not have a high fever and is not systemically ill. A 3-day course of IM ceftriaxone is as effective as a 10-day course of amoxicillin. High-dose amoxicillin is recommended for patients younger than 2 years of age, who are in day care, or those with recent exposure to antibiotics. Auralgan, a mixture of benzocaine and antipyrine, is a local anesthetic that may provide some direct analgesia, but should not be used in patients with tympanic membrane perforation.

79. Which of the following is true regarding neck trauma? (A) Delayed neurologic deficits after blunt neck trauma suggest carotid artery dissection. (B) All patients suspected to have an esophageal injury should receive a barium contrast esophagram. (C) Zone III injuries are most amenable to surgical exploration. (D) All neck wounds should be probed to determine the depth of the wound and integrity of vital structures. (E) Impaled objects should always be removed in patients with penetrating neck trauma.

79. Answer A. Delayed presentation of neurologic deficits is characteristic of vascular injuries to the neck due to blunt trauma. Only 10% of patients who ultimately develop neurologic deficits due to vascular injury exhibit signs and symptoms of injury within the first hour. Most patients experience stroke symptoms such as hemiparesis, hemiplegia, or aphasia between 1 and 24 hours after injury due to carotid artery dissection or thrombosis. Furthermore, a significant percentage of patients will not develop symptoms until after 24 hours has passed. Vertebral artery injury can also occur, although it is less common. Because the vertebral arteries combine to form a single basilar artery, injuries to the vertebral artery do not produce lateralizing symptoms. Patients may present with nausea, vomiting, central vertigo, and visual changes. Esophageal injuries are relatively uncommon, frequently subtle, and often missed in the setting of neck trauma. Because barium may provoke an inflammatory mediastinitis, patients with suspected esophageal injury should first receive an examination with water-soluble contrast such as gastrografin. If the initial study is negative, a follow-up examination with barium can be performed because of its superior sensitivity for smaller defects. Zone II injuries are most amenable to surgical repair due to the relatively uncomplicated surgical exposure and vascular control. Neck wounds should never be probed outside the operating room. If careful examination fails to determine platysma integrity, a surgeon should be consulted for presumed violation of the platysma. Impaled objects should always be left in place because they may tamponade vascular injuries. They should only be removed in the operating room under direct visualization.

8. Which of the following is the most common complication of otitis media? (A) Tympanic membrane (TM) perforation (B) Hearing loss (C) Labyrinthitis (D) Meningitis (E) Brain abscess

8. Answer B. All of the choices are complications of otitis media, but hearing loss is the most common. The primary reason for antibiotic treatment of otitis media is the prevention of these complications. Current treatment guidelines by the American Academy of Pediatrics involve both immediate antibiotic therapy and deferment of therapy with watchful waiting for 48 hours to observe for development of complications.

80. Which of the following is the most important factor in determining the chance of spontaneous passage of a kidney stone? (A) Composition of the stone (B) Size of the stone (C) Degree of pain (D) Degree of nausea (E) Age of the patient

80. Answer B. The large majority of kidney stones <5 mm will pass spontaneously without the need for lithotripsy or surgical extraction. The large majority of stones >5 mm will not pass spontaneously. Stones may become stuck in several areas along the urinary tract—renal calyx, ureteropelvic junction, midureter at the iliac vessels, ureterovesicular junction, and vesical opening. Urologic management of kidney stones is through extracorporeal shock wave lithotripsy, percutaneous nephrolithotomy, or surgical extraction. Pain and nausea are potential indications for admission of patients with kidney stones but are not independently associated with low passage rate. Age of the patient and composition of the stone are not directly associated with passage of the stone.

81. A 70-year-old woman presents with blurry vision in her right eye and a right-sided headache. The patient also complains of malaise, and is tender in her right temporal region. Which of the following is true regarding this patient's illness? (A) Steroids are ineffective in preventing disease in the contralateral eye. (B) Patients have a higher risk of arterial aneurysms. (C) Steroid therapy should be withheld until definitive diagnosis is made. (D) Men are more commonly affected. (E) The disease is very rare in patients older than 50 years of age.

81. Answer B. The patient has temporal arteritis. Steroids are the mainstay of management and should be initiated even before temporal artery biopsy is done. Once significant visual loss has occurred, only one out of three patients who receive IV steroids will improve in the affected eye. However, steroids will nearly eliminate contralateral eye involvement. Patients have a significantly higher risk of thoracic aorta aneurysms, and a slightly higher risk of abdominal aorta aneurysms. Women are five times more likely to be affected than men. Patients younger than 50 years are rarely affected.

89. A 68-year-old man with a history of chronic obstructive pulmonary disease (COPD) presents to the emergency department (ED) with worsening dyspnea, cough, and subjective intermittent fevers. He tells you that he spent 1 week in the intensive care unit (ICU) 6 months ago after being intubated for a similar episode and states "I don't ever want to be intubated again." You discuss the use of noninvasive positive pressure ventilation (NIPPV) with him (bilevel positive airway pressure [BiPAP]) and he is agreeable. With which of the following comorbidities is BiPAP safe to use? (A) Excessive secretions (B) Decreased sensorium (C) Severe hypertension (D) Midfacial trauma (E) Uncooperative patient

89. Answer C. NIPPV has revolutionized the treatment of COPD, cardiogenic pulmonary edema, as well as neuromuscular disease (e.g., myasthenia gravis). Contraindications for NIPPV include noncompliance (which is the most common reason for treatment failure), midfacial trauma (preventing an appropriate fit for the mask), excessive secretions or retention of secretions, decreased sensorium with absent cough and pharyngeal reflexes, recent gastric surgery (because of possible gastric distension) and vasopressor-dependent hypotension. Hypertension does not affect, nor is it affected by NIPPV.

82. A 4-year-old previously healthy girl presents to the ED with a 2-day history of cough productive of yellow sputum, fevers up to 102.4°F, pleuritic chest pain, and dyspnea. On examination, she is tachypneic and mildly toxic appearing. Her chest x-ray reveals a right middle lobe infiltrate. Which of the following organisms is the most likely cause of this problem? (A) Klebsiella pneumoniae (B) Mycoplasma pneumoniae (C) Chlamydia trachomatis (D) Respiratory syncytial virus (RSV) (E) Streptococcus pneumoniae

82. Answer E. S. Pneumoniae (also known as Pneumococcus) is the most common bacterial cause of pneumonia in preschool-aged children (6 months to 5 years). Overall, viruses are the most common pathogens causing pneumonia in this age group, with RSV being the most common, followed by parainfluenza and influenza viruses, as well as adenovirus and rhinovirus. However, this patient did not present with a viral prodrome and is mildly toxic upon examination. It is critical, therefore, to treat this patient with antibiotics that target S. Pneumoniae. Because of increasing resistance amongst S. Pneumoniae isolates, high-dose amoxicillin is the drug of choice, although patients who are hospitalized may require ampicillin, cefuroxime, or cefotaxime delivered intravenously. K. pneumoniae is an uncommon cause of pneumonia in children though it can cause severe infections in immunocompromised hosts. Community-acquired Klebsiella is primarily a disease of debilitated older men with a history of alcoholism. M. pneumoniae is the most common pathogen causing pneumonia in children aged 5 to 15 years. C. trachomatis may cause pneumonia in infants aged 3 weeks to 3 months, typically causing an afebrile, subacute interstitial pneumonia. RSV bronchiolitis and pneumonia are the primary causes for hospitalization during the first year of life.

84. The family of a 49-year-old woman with a history of hypertension brings her to the emergency department (ED) with a chief complaint of mumbled, incomprehensible speech that started approximately 5 hours ago along with weakness of her right arm, leg, and face. Computed tomography (CT) of her head reveals an area of infarction in her left hemisphere but no evidence of bleeding. Upon returning from CT, the nurse tells you her blood pressure is 200/105. Which of the following summarizes the best approach to managing this patient's blood pressure? (A) Start a sodium nitroprusside drip and titrate to a systolic blood pressure (SBP) of 160 mm Hg. (B) Give the patient her oral antihypertensive medications at her usual doses. (C) Administer 5 to 10 mg of labetalol IV every 10 to 20 minutes until the patient's SBP is between 140 and 160 mm Hg. (D) Give the patient 60 mg of nimodipine PO because of its dual effects in lowering blood pressure and in preventing vasospasm. (E) Continue to monitor the patient's blood pressure without treatment.

84. Answer E. Because cerebral perfusion pressure (CPP) = mean arterial pressure (MAP) - intracranial pressure (ICP), acute reductions in the MAP may have drastic and potentially grave effects on a patient's CPP. One of the most common errors in the treatment of patients with ischemic strokes is the overly aggressive treatment of hypertension. Because this patient is not a candidate for tPA (because she presented >3 hours after the onset of her symptoms), the general consensus is that her hypertension does not warrant treatment unless her blood pressure is >220/120 mm Hg or her MAP is >130 mm Hg. If this is the case, then either nitroprusside, labetalol, or IV enalapril may be used to rapidly gain control of the patient's blood pressure. Additionally, in patients who are tPA candidates, these medicines can be used to bring a patient's blood pressure below the 185/105 mm Hg threshold required for the administration of tPA. Exceptions to this approach may also apply in patients with ischemic stroke and concomitant MI, aortic dissection, or acute renal failure due to malignant hypertension. Such patients may require emergent interventions to decrease blood pressure due to these concomitant conditions. In all other patients with acute ischemic stroke, moderate hypertension as defined in the preceding text is thought to be neuroprotective by maintaining adequate CPP. Nimodipine is a calcium antagonist that is used in patients with SAH, and it has been shown to improve outcome in such patients. Whether it exerts its protective effects by reducing vasospasm is still an object of study.

85. A 12-year-old boy presents with progressive testicular swelling. Testicular examination is shown in Figure 1-15. Which of the following is the most common complication of this condition? (A) Testicular torsion (B) Epididymitis (C) Infertility (D) Malignancy (E) Deep venous thrombosis

85. Answer C. The patient has a large left-sided varicocele. Varicoceles are caused by abnormal dilation of the testicular vein and pampiniform plexus of the scrotum due to venous pooling from impaired drainage of the left internal spermatic vein into the left renal vein. Large varicoceles substantially increase the risk of infertility due to impaired blood flow and temperature of the ipsilateral testis. Testicular torsion may occur in patients with varicoceles but is not as common as infertility. Epididymitis and malignancy do not occur at appreciably higher rates in patients with varicoceles. Deep venous thrombosis due to inferior vena cava thromboses may cause varicoceles, but this is rare. (Figure from Fleisher GR, Ludwig S, Henretig FM, et al. eds. Textbook of pediatric emergency medicine, 4th ed. Philadelphia: Lippincott Williams & Wilkins; 2000, with permission.)

86. Which of the following is true regarding imaging of patients with suspected sinusitis? (A) Plain films are more accurate in diagnosing frontal and ethmoid than maxillary sinusitis. (B) Water's view plain film is the most sensitive test for maxillary sinusitis. (C) Computed tomography (CT) scans are able to differentiate between acute bacterial and viral sinusitis. (D) CT scans are both highly sensitive and highly specific in the diagnosis of sinusitis. (E) Most patients with viral upper respiratory infections (URIs) have abnormal CT scan findings.

86. Answer E. Sinus imaging has very little role in the ED except in the evaluation of patients with suspected complications of sinusitis (e.g., orbital cellulitis, cavernous sinus thrombosis). However, in the modern era it is clear that plain films have such a low sensitivity for sinus disease that they are virtually never requested. When plain films are used, they are best used for the diagnosis of maxillary sinusitis as their sensitivity drops when used to image other sinuses. CT scans are currently the most useful imaging modality in the evaluation of sinusitis. Although they are highly sensitive tests, they have a poor specificity (high false positive rate). Several studies have demonstrated that asymptomatic patients with uncomplicated viral URIs have abnormalities of one or both maxillary sinuses on CT scan. The same is true for asymptomatic patients with seasonal allergies. Therefore, CT scans should never stand alone in the diagnosis of sinusitis. A diagnosis of sinusitis should be supported with clinical findings of facial or dental pain, headache, and purulent nasal discharge, often preceded by a viral URI.

87. Among patients presenting to the ED with chest pain, which of the following physical examination findings is the biggest risk factor for aortic dissection? (A) Severe obesity (BMI >40) (B) Crescendo-decrescendo systolic ejection murmur at the right upper sternal border (C) Long, slender fingers (D) A systolic blood pressure difference of 15 mm Hg between the upper extremities (E) A very muscular physique

87. Answer C. Hypertension is the most common risk factor for aortic dissection though it is present in <75% of patients with diagnosed aortic dissection. Young patients with aortic dissection are much less likely to have hypertension (only approximately one-third of patients under 40 have hypertension). Marfan's syndrome is a connective tissue disorder characterized by several skeletal, cardiovascular, and ocular manifestations. Arachnodactyly (long, slender fingers) is one of the major diagnostic criteria and one of the most easily recognized manifestations of the disease. Other major manifestations include a short trunk with long legs, pectus carinatum or excavatum, reduced elbow extension, and significant scoliosis. While an upper extremity blood pressure disparity is a classically described finding in patients with aortic dissection, it is unreliable and should be at least 20 mm Hg. A pulse deficit between the upper extremities is a more reliable sign of underlying dissection but is present in only 15% of patients. The classic heart murmur in aortic dissection is the murmur of aortic regurgitation, which is present in up to onethird of patients and more than 40% of patients with type A dissections. Obesity is not a risk factor for aortic dissection. Power lifting, or very intense weightlifting, can cause transient elevations of intra-aortic pressure to more than 300 mm Hg and is thought to be a risk factor for dissection. Other risk factors include cocaine use, third trimester pregnancy, and other connective tissue disorders such as that found in Turner syndrome or Ehlers-Danlos syndrome.

9. Which of the following is true regarding isopropanol poisoning? (A) Blood urea nitrogen (BUN) may be falsely elevated. (B) Ketosis without acidosis is the characteristic lab abnormality. (C) Elevated anion gap is seen in most patients. (D) Ocular accumulation causing blindness is the typical pathophysiologic finding. (E) Fomepizole should be administered empirically in patients with high suspicion of isopropanol poisoning.

9. Answer B. Most ingested isopropanol is converted to acetone by hepatic alcohol dehydrogenase. Acetone is a ketone body, but is not acidic or charged, and does not contribute to the anion gap. Ketosis without acidosis is the characteristic finding in patients with isopropanol poisoning. Serum creatinine, not BUN, may be falsely elevated because of laboratory interference by acetone. Choice D is seen with methanol poisoning. Fomepizole is not indicated, as the metabolites of isopropanol, like the parent compound, cause generalized central nervous system (CNS) depression without other major organ system effects. Dialysis may be indicated to treat extremely severe poisoning.

90. You are working in the ED when a 74-year-old woman with chronic obstructive pulmonary disease (COPD) presents with an acute COPD exacerbation. You start her on bilevel positive airway pressure (BiPAP) at a rate of 10, an inspiratory positive airway pressure (IPAP) of 10 mm Hg and an expiratory positive airway pressure (EPAP) of 4 mm Hg. Twenty minutes later, the patient's oxygenation has not improved. Which of the following changes would most likely increase this patient's oxygenation? (A) Increase the patient's IPAP from 10 to 15 (B) Increase the patient's rate from 10 to 12 (C) Decrease the patient's EPAP from 4 to 2 (D) Increase the patient's EPAP from 4 to 7, and the IPAP from 10 to 15 (E) Decrease the patient's EPAP from 4 to 2, and decrease the IPAP from 10 to 5

90. Answer D. BiPAP ventilation emulates pressure support mechanical ventilation with positive end-expiratory pressure (PEEP) in which the machine cycles between two different pressure levels during inspiration and expiration. This is in contrast to continuous positive airway pressure in which the pressure is the same throughout inspiration and expiration (i.e., IPAP = EPAP). When a patient's oxygenation status is not improving, the two main adjustments that can be made are to increase the inspired FIO2 and to increase the PEEP. With BiPAP, there is a limit to the amount of oxygen that can be applied because the oxygen is not being delivered to the lower airways, as is the case with mechanical ventilation. However, the EPAP (i.e., PEEP) can be increased in order to recruit additional alveoli (by stenting them open with increased pressure at the end of expiration), which increases the surface area for gas exchange and hopefully results in improved oxygenation. In order to maintain the PPD during inspiration, the IPAP may be concomitantly increased (to ensure ongoing adequate ventilation). If these measures fail, the patient requires intubation and mechanical ventilation.

91. Which of the following is a manifestation of hypocalcemia? (A) QTc shortening (B) Polyuria (C) Perioral paresthesias (D) Nephrolithiasis (E) None of the above

91. Answer C. The most common symptoms of hypocalcemia are neurologic and generalized irritability such as twitching and paresthesias which may progress to frank tetany, perioral paresthesias, as well as Chvostek's and Trousseau's signs. Chvostek's sign is a twitch of the upper lip when the area around the facial nerve is tapped. Trousseau's sign is carpopedal spasm when a blood pressure cuff is inflated over the arm at greater than systolic blood pressure for longer than 3 minutes. The other signs and symptoms listed are all manifestations of hypercalcemia.

92. A 62-year-old woman with a history of hypertension, diabetes mellitus, and emphysema presents with a 3-week history of dyspnea on exertion, three-pillow orthopnea, and bilateral lower extremity edema. She denies fevers, cough, or chest pain. A chest x-ray is shown in Figure 1-16A and B. The patient's electrocardiogram (EKG) does not show any ischemic changes. Her vitals are: Respiratory rate 22 per minute, pulse 108, BP 154/88, pulse oximetry 88% on room air. What is the next best step in management? (A) Perform an immediate therapeutic thoracentesis (B) Perform a diagnostic thoracentesis to rule out empyema (C) Administer oxygen, nitrates, and furosemide (D) Perform rapid sequence intubation (E) Order a B-type natriuretic (BNP) level in order to determine the best management.

92. Answer C. This patient has an exacerbation of congestive heart failure (CHF). Although she has no prior diagnosis of CHF, her clinical history is consistent with the diagnosis. Pleural effusions are common in CHF and thoracentesis generally has no role in their management because they typically resolve with diuretics. This patient also lacks infectious symptoms, which would suggest the presence of pneumonia or a parapneumonic effusion. In patients with severe respiratory compromise as a result of large pleural effusions in the setting of CHF, a secure airway through rapid sequence intubation would be the first step. Although this patient is hypoxic, she has not yet been given supplemental oxygen, so it is premature to intubate this patient at this point. Furthermore, noninvasive ventilation might be attempted before establishing a secure airway in patients with only mild to moderate respiratory distress. BNP levels may be helpful in determining the etiology of respiratory distress in patients who have mixed cardiopulmonary disease without a clear clinical picture. This patient's clinical picture clearly points toward a diagnosis of CHF, so a BNP level would not provide any additional information.

93. An ED thoracotomy is performed on a trauma patient. Which of the following is the structure labeled in Figure 1-17? (A) Vagus nerve (B) Phrenic nerve (C) Sympathetic chain (D) Spinal accessory nerve (E) Inferior vena cava

93. Answer B. Emergency thoracotomy is indicated in patients who have traumatic arrest in the ED or shortly before arrival from penetrating thoracic trauma. The initial incision of the chest wall begins in the sternal area and sweeps along the superior border of the rib all the way laterally to the edge of the bed. The rib spreaders are then placed and the left lung is moved out of the way to expose the pericardium. An incision to the pericardium should be made anterior to the prominent phrenic nerve, which is visible in the figure. Damage to the phrenic nerve may cause diaphragmatic weakness and seriously impair respiratory function. (Figure courtesy of Mark Silverberg, MD. Reprinted with permission from Silverberg M. Greenberg Text-Atlas of Emergency Medicine. Lippincott Williams & Wilkins; 2004:22.)

94. Primary chronic adrenal insufficiency is usually due to: (A) Sarcoidosis (B) Hemorrhage (C) Pituitary insufficiency (D) Idiopathic (E) Iron deposition

94. Answer D. Increasingly, acute adrenal insufficiency is thought to be a rare condition, and most cases of acute adrenal insufficiency probably represent an exacerbation of chronic disease. When acute adrenal insufficiency occurs, it is most commonly due to exogenous glucocorticoid administration. Chronic adrenal insufficiency is idiopathic (thought to be autoimmune-mediated destruction of the adrenal gland) in 66% to 75% of cases.

95. A 34-year-old man with schizophrenia is brought to the ED by his family because he "keeps ignoring" them. They report that for the last few hours, when they argue with him about taking his risperidone, he stares off into the space and does not acknowledge them in any way. This lasts for a few minutes and gradually improves until the next conversation. They want to speak with the psychiatrist about putting him on a new antipsychotic medication. His vital signs are normal and physical examination is unremarkable except that when you question him about his medication, he becomes visibly angry and his eyes look up to the ceiling. After he calms down in a few minutes, he resumes normal eye contact and conversation. Which of the following is the most appropriate action at this time? (A) Discharge home with outpatient psychiatry follow-up (B) Consult psychiatry for alprazolam prescription (C) Haloperidol 5 mg IM (D) Lorazepam 2 mg IM (E) Benztropine 1 mg IM

95. Answer E. The patient appears to have evidence of a specific dystonic reaction known as the oculogyric crisis, where both the patient's eyes stare upward and do not come back to the neutral position. Symptoms can fluctuate based on emotional state. Dystonic reactions are usually due to an excess of cholinergic activity due to overblockade of dopaminergic receptors by antipsychotic medications. The normally inhibitory effect of dopamine on the cholinergic neurons is reduced with the use of antipsychotic agents. Treatment for acute dystonic reactions is with an anticholinergic agent, either benztropine or diphenhydramine. Discharging the patient home neither treats the patient nor adequately manages the social situation. Psychiatric consultation for a new psychotropic medication prescription is not appropriate until the primary cause of the complaint has been evaluated and addressed. Haloperidol would further worsen the oculogyric crisis. Lorazepam might sedate the patient but would probably not improve the dystonia.

96. A 27-year-old man presents to the ED with pruritus ani, tenesmus, and yellowish mucoid discharge from his rectum. Upon further questioning, he acknowledges recent unprotected anal intercourse. The recommended regimen for treating this patient is: (A) Topical podophyllin b.i.d. × 7 days (B) Ceftriaxone 125 mg IM plus doxycycline 100 mg PO b.i.d. × 7 days (C) Valacyclovir 1 g PO daily × 5 days (D) Benzathine penicillin G 2.4 million units IM × 1 dose (E) Lopinavir

96. Answer B. Any patient with a history of recent unprotected anal intercourse who presents with symptoms of proctitis should be treated empirically for N. gonorrhoeae proctitis. Because concurrent infection with Chlamydia trachomatis is common in patients infected with gonorrhea, empiric therapy should cover this organism as well. Podophyllin is a treatment for human papillomavirus (condyloma acuminatum). Valacyclovir is a treatment for herpes proctitis, penicillin is a treatment for syphilis, and lopinavir is a protease inhibitor used to treat human immunodeficiency virus (HIV). Owing to this patient's high-risk lifestyle, he should undergo testing for HIV and for syphilis but empiric treatment is not necessary.

97. An 18-year-old primigravida at 35 weeks of gestation presents with abdominal pain, vaginal discharge, and a low-grade fever. She states that she was well until two days ago when, after coughing, she experienced a small stream of fluid running down her leg which she thought was urine. Her temperature is 102°F and her pulse is 108. Her examination reveals a yellowish discharge in the vaginal vault with a tender uterus. What is the most likely cause of her symptoms? (A) Abruptio placentae (B) Pelvic inflammatory disease (C) Bacterial vaginitis (D) Pyelonephritis (E) Chorioamnionitis

97. Answer E. This patient experienced preterm PPROM 2 days before presentation, which dramatically increased her risk for developing chorioamnionitis. Chorioamnionitis is an intra-amniotic infection that is most commonly due to vaginal flora that has gained entry to the amniotic cavity. Risk factors include young age, low socioeconomic status, nulliparity, extended duration of labor and ruptured membranes, multiple vaginal examinations, and preexisting infections of the lower genital tract. The typical causative organisms are group B streptococci and E. coli and the most widely used antibiotic regimen is a combination of ampicillin and gentamicin. Ampicillin may be replaced with vancomycin, erythromycin, or clindamycin in penicillin-allergic patients.

98. Which of the following parenteral agents is the initial preferred agent for blood pressure management in patients with acute aortic dissection? (A) Hydralazine (B) Enalapril (C) Labetalol (D) Diltiazem (E) Nicardipine

98. Answer C. Initial treatment should focus on reducing the patient's blood pressure to a systolic blood pressure of 100 to 120 mm Hg or lower if the patient can tolerate it using parenteral beta-blockers (labetalol, esmolol, propranolol). Beta-blockers are the preferred agent because they also reduce heart rate and aortic wall stress by reducing the rate of systolic blood pressure rise. The nonselective CCBs (verapamil and diltiazem) are reasonable alternatives in patients who are unable to tolerate beta-blockers (e.g., due to bronchospasm). If the patient's blood pressure remains elevated above this range despite maximal beta-blocker therapy, or if the patient's blood pressure is at goal, with normal mentation, urine output, and renal function, then the blood pressure can be further reduced using nitroprusside. Nitroprusside is not considered a first-line agent because its use triggers reactive tachycardia and increased contractility. Thus, it is important to use it only in patients who have first achieved effective beta-blockade. As a direct arterial vasodilator, hydralazine should be avoided because its use results in reflex tachycardia and it has an unpredictable blood pressure response curve. Otherwise, each of the remaining agents may be used as a second-line agent after beta-blockade is initiated.

99. A 20-year-old man presents with fever, sore throat, and fatigue for 1 week. He took a friend's antibiotic after 2 days of illness and developed a red rash all over his body, so he stopped the antibiotic. His oropharynx is mildly red without exudates, and he has cervical lymphadenopathy and splenomegaly. Rapid strep testing and monospot are both negative. Which of the following is the most likely diagnosis? (A) Group A beta-hemolytic streptococcal pharyngitis (B) Infectious mononucleosis (C) Influenza (D) Herpangina (E) Diphtheria pharyngitis

99. Answer B. The patient has clinical signs and symptoms of infectious mononucleosis, usually due to Epstein-Barr virus. The sensitivity of the monospot test is less than 70% during the first week of illness and only about 80% during the second week. Therefore, a negative monospot test in the first 2 weeks of mono does not rule out the diagnosis. Group A beta-hemolytic streptococcal pharyngitis usually causes tonsillar exudates and rarely causes splenomegaly. Influenza generally causes a mild pharyngitis without splenomegaly. Herpangina is due to coxsackievirus and causes painful vesicular lesions on the soft palate and tonsils. Diphtheria causes pharyngitis in unvaccinated individuals with a characteristic, grayish-green pseudomembrane that forms when the exudate coalesces. Stridor and frank airway obstruction can form in these cases.

6. Which of the following is the earliest electrocardiogram (EKG) finding in acute myocardial infarction (MI)? (A) Hyperacute T waves (B) ST elevation (C) ST depression (D) T-wave inversion (E) Q waves

Answer A. The temporal sequence of EKG morphologies in acute MI is generally hyperacute T waves, ST elevation, T-wave inversion, and Q waves. Hyperacute T waves may also be seen in hyperkalemia and (along with ST elevation) in benign early repolarization, acute pericarditis, and left ventricular hypertrophy. ST elevation may also be seen in bundle branch block and ventricular aneurysms. The evolution in the preceding sequence in the appropriate clinical setting usually points to STEMI. This is a strong argument for performing repeated EKGs in the evaluation of acute coronary syndrome in the ED.

A 34-year-old man is brought into the emergency department (ED) after a motor vehicle collision. Which of the following findings is an indication to perform a computed tomography (CT) of the abdomen/pelvis with IV contrast to look for renal injury in this patient? (A) Microscopic hematuria (B) Gross hematuria (C) Flank pain (D) Flank ecchymosis (E) Penile hematoma

Answer B. In adult blunt trauma patients, the indications to perform an evaluation for renal injury include gross hematuria, microscopic hematuria plus hypotension, or significant deceleration injury. In the absence of shock, microscopic hematuria is not an indication for CT in blunt trauma patients. However, microscopic hematuria may also be a marker of urethral or bladder injury and may warrant further investigation depending on the mechanism of injury and findings on physical exam. The presence of flank physical examination findings is not indicative of renal injury significant enough to warrant imaging. Penile involvement can certainly cause urethral damage, but it does not confer a higher risk of renal injury.

Which of the following represents the characteristic acid-base disturbance seen in aspirin toxicity? (A) Respiratory alkalosis alone (B) Metabolic acidosis alone (C) Mixed respiratory alkalosis with metabolic acidosis (D) Mixed respiratory acidosis with metabolic alkalosis (E) No acid-base abnormality

Answer C. Aspirin (acetyl-salicylic acid) causes a mixed acid-base disorder. It causes a metabolic acidosis by uncoupling oxidative phosphorylation which leads to a build up of organic acids, and it causes a respiratory alkalosis by stimulating the medullary respiratory centers which results in hyperventilation. The net effect on pH varies, but generally results in acidosis in children and alkalosis in adults.


Kaugnay na mga set ng pag-aaral

Us History 2- Chapter 22-24 Test Review

View Set

Electrical Motor Controls, Chapters 21-30 Tech-Cheks

View Set

B Intro to IT unit 5 variables and data types assignment

View Set